You are on page 1of 151

1

Chromatography Course Dr Ehab Aboueladab-Lecturer of Biochemistry-Mansoura University-Branch Damietta




Chapter One
Aim Separation Techniques
1-Biological fluids are extremely complex in composition.
2-Chemical analysis would be impossible if it were necessary to completely
isolate each substance prior to its measurement.
3- An optimal method tests for a specific substance in the presence of all
others, requiring no isolation of the substance under analysis.
4- A test is specific when none of the other substances present interfere.
However, virtually all chemical tests are subject to at least some
interference.
5-This is one of the most important problems in clinical chemistry. Therefore
some type of separation procedure is required.
7-Separation in clinical chemistry usually is based on differences in the
size, solubility or charge of the substances involved.

Introduction
Chromatography is a separation method in which the analyte is
contained in a mobile phase and pumped through a stationary phase.
Sample components interact differently with these two phases and elute
from the column at different retention times t
R
. Since the first description
of chromatography by Russian botanical scientist Mikhail Semenovich
Tswett is discovery of chromatography. He used a column of powdered
calcium carbonate to separate green leaf pigments into a series of
colored bands by allowing a solvent to percolate through the column bed.
Since these experiments by Tswett many scientists have made substantial
contributions to the theory and practice of chromatography. Not least among


2
Chromatography Course Dr Ehab Aboueladab-Lecturer of Biochemistry-Mansoura University-Branch Damietta


these is A. J. P. Martin who received the Nobel Prize in 1952 for the
invention of partition chromatography (with R. L. K. Synge) and in the
same year with A. T. James he introduced the technique of gas-liquid
chromatography. Chromatography is now an important tool used in all
branches of the chemical and life sciences.
Chromatographic separations can be described quantitatively with a number
of parameters including the capacity factor k , the selectivity factor , the
plate number N or height equivalent of a theoretical plate H and the
resolution RS. The optimum flow rate of a chromatographic separation can
be determined with the van Deemter equation. In bioanalytical chemistry,
chromatography is mainly employed for the isolation and purification of
proteins. Reversed phase chromatography can separate biomolecules
according to their interaction with the hydrophobic stationary phase and
the hydrophilic moblile phase. This separation method can be coupled to
an ESI mass spectrometer. Ion exchange chromatography separates
molecules depending on their net charge. Affinity chromatography makes
use of molecular recognition between biomolecules; and size exclusion
chromatography allows for the separation of molecules depending on
their size.
1-Definition of Chromatography
Chromatography is essentially a physical method of separation in
which the components to be separated are distributed between two phases
one of which is stationary (stationary phase) while the other (the mobile
phase) through it in a definite direction.



3
Chromatography Course Dr Ehab Aboueladab-Lecturer of Biochemistry-Mansoura University-Branch Damietta


Chromatography is a technique for separating a sample into various
fractions. The heart of any chromatography is the stationary phase (liquid or
solid). The stationary phase is attached to a support, a solid inert material.
The sample (gas, liquid or solid which may or may not dissolved in solvents)
is carried across the stationary phase by a mobile phase (gas or liquid). The
sample components undergo a series of exchanges (partitions) between the
two phases due to the differences in their chemical and physical properties.
These differences govern the rate of movement (called migration) of the
individual components.
2- Classification of chromatographic methods
The common feature of all chromatographic methods is two phases,
one stationary and the other mobile
A classification can be made depending upon whether the stationary phase
is solid or liquid. If it is solid, the method is termed adsorption
chromatography; if it is liquid the method is partition chromatography.
One of the two phases is a moving phase (the mobile phase), while the
other does not move (the stationary phase). The mobile phase can be
either a gas or a liquid, while the stationary phase can be either a liquid
or solid.
3- Classification scheme
One classification scheme is based on the nature of the two phases.
All techniques which utilize a gas for the mobile phase come under the
heading of gas chromatography (GC). All techniques that utilize a liquid
mobile phase come under the heading of liquid chromatography (LC).
Additionally, we have gasliquid chromatography (GLC), gassolid


4
Chromatography Course Dr Ehab Aboueladab-Lecturer of Biochemistry-Mansoura University-Branch Damietta


chromatography (GSC), liquidliquid chromatography (LLC), and liquidsolid
chromatography (LSC),
4- Main Type of Chromatography
In general, there are four main types which can be classified as
follows:
4.1-Liquid-Solid chromatography
Classical adsorption chromatography (Tswett column)
Ion-exchange chromatography
4.2. Gas-Solid chromatography
4.3. Liquid-Liquid chromatography
Classical partition chromatography
Paper chromatography
4.4 Gas-Liquid chromatography
5-Separation techniques
Technique Property Description
Precipitation Solubility Some of the substances
precipitate while the others
remain dissolved
Ultra-filtration or Dialysis Molecular size Some of the substances
pass through a layer or
sheet of porous material
while the other substances
are retained
Extraction Solubility Some of the substances
dissolve (partition) more in


5
Chromatography Course Dr Ehab Aboueladab-Lecturer of Biochemistry-Mansoura University-Branch Damietta


water. While other
substances dissolve more
organic solvent in contact
with the water
Thin layer
Chromatography
or
Column Chromatography
Solubility Some of the substances
dissolve (partition) more in
the immobile file of water on
a solid supporting medium
(or stick more to the
exposed areas of the solid
supporting medium) while
the other substances
dissolve more in the
surrounding film of flowing
organic solvent
Gas liquid
Chromatography
Solubility Some of the substances
dissolve more in the
immobile film of wax or oil-
like material on a solid
supporting medium. While
the others dissolve more in
surrounding stream of
flowing gas.
Gel filtration
Chromatography
Molecular Size Some of the substances
diffuse into the pores in a


6
Chromatography Course Dr Ehab Aboueladab-Lecturer of Biochemistry-Mansoura University-Branch Damietta


porous, solid material while
others remain outside in the
surrounding stream of
flowing water
Ion-exchange
Chromatography
Electrical
charge
Some of the substances are
bound by immobile charges
on the solid supporting
medium while others are not
bound
Electrophoresis
Chromatography
Electrical
charge
The substances with more
charge move faster and,
therefore, further.
Substances with opposite
charges move in opposite
directions.

6-Adsorption chromatography
Adsorption column chromatography is the oldest form of
chromatography. Whether two or more substances of a mixture can be
separated by adsorption chromatography depends on a number of factors.
Most important is the strength with which each component of mixture is
adsorbed and its solubility in the solvent used for elution. The degree to
which a particular substance is adsorbed depends on the type of bonds
which can be formed between the solute molecules and the surface of the
adsorbent.


7
Chromatography Course Dr Ehab Aboueladab-Lecturer of Biochemistry-Mansoura University-Branch Damietta




5-Purpose of Chromatography
Analytical - determine chemical composition of a sample
Preparative - purify and collect one or more components of a sample
Other classification of Chromatographic Methods
Chromatography is classified according to three ways:


8
Chromatography Course Dr Ehab Aboueladab-Lecturer of Biochemistry-Mansoura University-Branch Damietta



1. According to the physical state of the mobile phase:
Liquid chromatography: This subdivided according to the stationary
phase into liquid-liquid or liquid-solid chromatography.
Gas chromatography: This subdivided according to the stationary
phase into Gas-liquid or Gas-solid chromatography.
2. According to the method of contact between the mobile phase and
stationary phase:
Column chromatography: the stationary phase is placed in a column
through which the mobile phase moves under the influence of gravity or
pressure. The stationary phase is either a solid or a thin, liquid film
coating on a solid particulate packing material or the columns walls.
Planar chromatography: the stationary phase coats a flat glass, metal, or
plastic plate and is placed in a reservoir containing the mobile phase
which moves by capillary action carrying with it the sample components
3. According to the chemical or physical mechanism responsible for
separating the samples constituents.(attractive forces)
Adsorption chromatography: for polar non-ionic compounds
Ion Exchange chromatography: for ionic compounds
Anion: analyte is anion; bonded phase has positive charge
Cation: analyte is cation; bonded phase has negative charge
Partition chromatography: based on the relative solubility of analyte in
mobile and stationary phases
Normal: analyte is non-polar organic; stationary phase MORE polar
than the mobile phase


9
Chromatography Course Dr Ehab Aboueladab-Lecturer of Biochemistry-Mansoura University-Branch Damietta


Reverse: analyte is polar organic; stationary phase LESS polar than
the mobile phase
Size Exclusion chromatography: stationary phase is a porous matrix.
6-The Principle of Chromatography
Chromatography is a separation method where the analyte is contained
within a liquid or gaseous mobile phase, which is pumped through a
stationary phase.
Usually, one phase is hydrophilic and the other lipophilic. The components of
the analyte interact differently with these two phases. Depending on their
polarity, they spend more or less time interacting with the stationary phase
and are thus retarded to a greater or lesser extend. This leads to the
separation of the different components present in the sample. Each sample
component elutes from the stationary phase at a specific time, its retention
time t
R
(Fig. 1.1). As the components pass through the detector, their signal
is recorded and plotted in the form of a chromatogram.
Chromatographic methods can be classified into
Gas chromatography (GC) and liquid chromatography (LC) depending on
the nature of the mobile phase involved.
Gas chromatography can be applied only to gaseous or volatile substances
that are heat-stable. The mobile phase, an inert carrier gas such as
nitrogen, hydrogen or helium, is pumped through a heated


10
Chromatography Course Dr Ehab Aboueladab-Lecturer of Biochemistry-Mansoura University-Branch Damietta



Figure1-1
column. This column can be packed with a silicon oxide based material or
is coated with a polymeric wax. The sample is vaporised, pumped through
the column and the analytes are detected in the gas stream as they exit the
column. Analyte detection can be achieved by either flame ionisation or
thermal conductivity. GC is not commonly used for the analysis of
biomolecules since large molecular weight compounds such as peptides and
proteins are thermally destroyed before evaporation. Smaller molecules such
as amino acids, fatty acids, peptides and certain carbohydrates can be
analysed if they are modified chemically to increase their volatility. Some cell
cultures produce volatile metabolites such as aldehydes, alcohols or
ketones. These can be analysed readily via GC.
In liquid chromatography, the sample is dissolved and pumped through a
column containing the stationary phase. LC is more versatile than GC as it is
not restricted to volatile and heat-stable samples; the sample only has to
dissolve completely in the mobile phase. Common detection methods are
UV spectroscopy, measurement of refractive index, fluorescence, electrical
conductivity and mass spectrometry.


11
Chromatography Course Dr Ehab Aboueladab-Lecturer of Biochemistry-Mansoura University-Branch Damietta


Modes of operation can be classified as normal and reversed phase
chromatography.
In normal phase chromatography, the stationary phase consists of a
hydrophilic material such as silica particles and the mobile phase is a
hydrophobic organic solvent such as hexane.
In reversed phase chromatography, on the other hand, the stationary
phase is hydrophobic and the mobile phase is a mixture of polar solvents, for
example water and acetonitrile. Biomolecules are generally soluble in polar
solvents; hence, reversed phase chromatography is the method of choice for
amino acids, peptides, proteins, nucleic acids and carbohydrates.
7-Comparison of classical and bioanalytical chemistry





12
Chromatography Course Dr Ehab Aboueladab-Lecturer of Biochemistry-Mansoura University-Branch Damietta


8-Application of Liquid Chromatography for Bioanalysis
In bioanalytical chemistry, chromatography is mainly used for the
separation, isolation and purification of proteins from complex sample
matrices. In cells, for example, proteins occur alongside numerous
other compounds such as lipids and nucleic acids. In order to be
analysed, these proteins must be separated from all the other cell
components. Then the protein of interest might have to be isolated from
other proteins and purified further. Chromatography is an essential part of
almost any protein purification strategy. A number of different
chromatographic techniques are used for the purification and analysis of
proteins. They can be classified according to the physical principle involved
in the separation process.
Typical examples include reversed phase chromatography, ion
exchange chromatography, affinity chromatography and size exclusion
chromatography (SEC) (Table 1.1). These are outlined in more detail in the
following sections.
Separation methods for proteins9- Basic Chromatographic Theory


13
Chromatography Course Dr Ehab Aboueladab-Lecturer of Biochemistry-Mansoura University-Branch Damietta


The optimisation of chromatography is aimed towards completely
separating all of the components of a sample in the shortest possible time.
This can, for example be achieved by modifying the composition of the
mobile phase, choosing a different stationary phase or by changing the flow
rate. A typical chromatogram is depicted in (Fig. 1.2).

Fig. 1.2. Definition of retention time, t
R
, and peak width, w.
The sample is injected into the chromatographic column at t = 0 s.
Substances that are not retarded by the stationary phase leave the column
at zero retention time, t0, corresponding to the flow rate of the mobile
phase. Compounds A and B are retarded by the stationary phase and leave
the column at their retention times t
R
(A) and t
R
(B), respectively. The peak
width, w, is defined as the intersection of the tangents on each side of the
peak with the baseline. These basic parameters, retention time and peak
width, can be used to derive a number of other parameters that express the
quality of the achieved chromatographic separation. In the following
paragraphs, a brief summary of the most important parameters of
chromatographic theory are discussed.
The capacity factor k' (equation1. 1) describes the velocity of the analyte
relative to the velocity of the mobile phase. Each compound spends a


14
Chromatography Course Dr Ehab Aboueladab-Lecturer of Biochemistry-Mansoura University-Branch Damietta


different amount of time interacting with the mobile and stationary phase.
The average velocity of a sample compound is dependent on how much time
it spends in the mobile phase. If k' is much smaller than 1, then the analyte
moves too quickly and the elution time is so short that an exact
determination of t
R
is difficult. If the sample moves too slowly, the separation
time is very high. A good value for k would be between about 1 and 5. The
selectivity factor (equation 1.2) describes the relative velocities of the
analytes with respect to each other. The selectivity describes how well a
chromatographic method can distinguish between two analytes.

capacity factor

(equation 1.1)
selectivity factor

(equation 1.2)

The efficiency of a chromatographic separation is dependent on band
broadening. If band broadening is large, peaks can overlap and resolution is
lost.
Band broadening for a column of length L is quantitatively expressed in the
concept of height equivalent of a theoretical plate, H, or simply plate
numbers, N (equations 1.3 and 1.4). The larger the number of plates N
and the smaller H is, the better the chromatographic efficiency.


15
Chromatography Course Dr Ehab Aboueladab-Lecturer of Biochemistry-Mansoura University-Branch Damietta



(equations 1.3)

(equations 1.4)
The parameters that influence band broadening can be approximated by the
van Deemter equation (equation 1.5) which is valid for gas and liquid
chromatography as well as capillary electrophoresis

(equations 1.5)
In this simplified equation, the height of theoretical plates, H, is given as a
sum of three terms. The first term, A, describes the influence of the column
packing on band broadening. This so-called Eddy diffusion is constant for
a given column and independent of the flow rate. The second term, B/u,
describes the diffusion in or opposed to the direction of flow. This
longitudinal diffusion is inversely proportional to the flowrate u. The third
term, Cu, describes the resistance to mass transfer between the stationary
and mobile phase which is directly proportional to the flow rate. By plotting H
as a function of u, the optimum flow rate for a chromatographic separation
can be determined (Fig. 1.3).


16
Chromatography Course Dr Ehab Aboueladab-Lecturer of Biochemistry-Mansoura University-Branch Damietta


Fig. 1.3. A van Deemter plot for the determination of the optimum flow rate.
The ultimate goal of a separation is to achieve a high resolution, Rs,
(equations 1.6 and 1.7). If Rs = 1.5, then peaks of identical area overlap by
only 0.3 %, an Rs = 1 equals a peak overlap of 4 %. Peak resolution can be
optimized by increasing the selectivity and minimizing band broadening.
Resolution

(equations 1.6)
valid for < 1.2
(equations 1.7)


17
Chromatography Course Dr Ehab Aboueladab-Lecturer of Biochemistry-Mansoura University-Branch Damietta



As can be seen from equation 1.7, the capacity factor k' has a great
influence on the resolution. Usually the components in the sample have a
wide variety of k values. If conditions are optimised such that the first
compounds to elute have k' values between the optimum of 1 and 5, then the
other compounds with higher k values elute much later and show excessive
band broadening. If, on the other hand, conditions are optimised for the later
eluting compounds, then the resolution will be poor for the compounds that
elute first. This general elution problem can be overcome by decreasing k'
during the separation. In LC, the composition of the mobile phase can be
changed during the separation. This is called a gradient elution as
opposed to an isocratic elution, where the composition of the mobile phase
remains unchanged during the separation process. In GC, a temperature
gradient can be applied during separation rather than operating under
isothermic conditions. Generally, the first step in trying to achieve a good
separation of the sample mixture is to choose a stationary phase with which
the analyte can interact. Then, the composition and gradient of the mobile
phase can be chosen to optimise the capacity factor and resolution.
Chromatographic theory as outlined in the above paragraphs can be
applied to the analysis of smaller molecules such as amino acids,
peptides and short biopolymers. Care has to be taken for larger
biomolecules such as high molecular weight proteins. These often show
different behavior and the theory can only be applied to a limited extent.





18
Chromatography Course Dr Ehab Aboueladab-Lecturer of Biochemistry-Mansoura University-Branch Damietta


10-Reversed Phase Liquid Chromatography
Normal phase chromatography was developed many years before reversed
phase chromatography was investigated. Initially, stationary phases were
made of polar materials such as paper, cellulose or silica gel and the
mobile phase consisted of non-polar solvents such as hexane or
chloroform. Only at a later stage were these phase polarities reversed. Polar
solvents such as water and acetonitrile were

Fig. 1.4. Surface groups used for stationary phases in reversed phase
chromatography range from ethyl silane (C2) to n-octadecyl silane (C18).
used in combination with non-polar stationary phases. These were obtained
by etherification of the polar hydroxyl groups of the silica gel with long alkyl
chains. Reversed phase chromatography is the method of choice for the
separation of smaller biomolecules such as peptides, amino acids,
carbohydrates and steroids, which are soluble in water/acetonitrile
mixtures. The separation of proteins can be problematic as organic solvents
such as acetonitrile can decrease the proteins solubility and cause
denaturisation.
The stationary phase usually consists of porous silica particles with non-
polar surface groups (Fig. 1.4), obtained from etherification of the initial


19
Chromatography Course Dr Ehab Aboueladab-Lecturer of Biochemistry-Mansoura University-Branch Damietta


hydroxyl groups of the silica particle with silanes containing non-polar
hydrocarbon chains. Any chain length from ethyl silane (C2) to n-octadecyl
silane (ODS) (C18) is used, although octyl silane (C8) and ODS are the most
commonly employed chain lengths. For analytical separations, the particle
size is typically 5 m or smaller. In preparative liquid chromatography, where
the goal is to isolate a compound of interest for further analysis or
investigation, larger particles with a higher capacity and larger column
diameters are used. The pore size of the silica particles is usually about 10
nm, resulting in a very large surface area, as much as 100 to 400 m
2
/g.
This gives the analytes ample opportunity to interact with the stationary
phase whilst flowing through the separation column.
The mobile phase is based on a polar solvent system consisting of an
aqueous buffer and acetonitrile or methanol. Gradient elution is often
employed to increase resolution and shorten separation times. This is
achieved by increasing the organic solvent and thus decreasing the mobile
phase polarity and the retention of less polar analytes during the separation
process. Solvents can be classified according to their elution strength and
polarity (Fig. 1.5).
Buffer systems based on ammonium acetate, phosphate or hydrogen
carbonate are usually added at concentrations of about 20 mM to adjust the
pH of the mobile phase to values between 2 and 8. Ion pairing reagents can
be used at low concentrations, typically 0.1%, to increase the hydrophobicity
of charged analytes. They


20
Chromatography Course Dr Ehab Aboueladab-Lecturer of Biochemistry-Mansoura University-Branch Damietta



Fig. 1.5. Solvents ordered according to polarity and elution speed of the
analytes.

Fig. 1.6. Instrumental setup of an HPLC gradient system.
form ion-pair complexes with the analyte. Anionic ion pairing reagents such
as trifluoroacetic acid (TFA) bind to positively charged analytes,
whereas cationic ion pairing reagents such as tetra alkyl ammonium salts
can be used to bind to negatively charged analytes. These complexes are
retarded more by the stationary phase and are thus easier to separate than
the largely unretained charged analytes alone.
In modern chromatography, the separation columns are tightly packed with
small particles of about 15m in diameter. To achieve ambient flow rates in
these columns, high pressures of up to 300400 bars must be generated. A
typical instrumental setup for this high pressure or high performance liquid
chromatography (HPLC) is shown in Fig. 1.6.


21
Chromatography Course Dr Ehab Aboueladab-Lecturer of Biochemistry-Mansoura University-Branch Damietta


Computer controlled pumps move the mobile phase through the system.
Aqueous solvent A and organic solvent B are mixed to the desired
composition. In the case of gradient elution, the composition is gradually
altered during the separation.
Sample volumes are injected with either a manual loop and valve system or
automatically via an auto sampler. Depending on the column dimensions
sample volumes can be as low as several nL and as high as a mL. Often the
column is situated inside an oven which is thermostatically regulated to
maintain a constant temperature. After eluting from the column, the analytes
pass through the detector.
UV detection using a fixed wavelength could be performed at = 210 nm
for peptides and = 254 nm or = 280 nm for proteins. More expensive
instruments have diode array detectors (DAD) which can take several
whole spectra per second and allow for more unambiguous identification.
High sensitivity can be achieved via fluorescence detection of derivatised
amino acids and peptides. A more recent development is to couple
liquid chromatography systems to an electrospray ionisation mass
spectrometer, ESI-MS.
Mass spectrometry allows universal detection at very high sensitivity and
also gives structural information about the analyte. However, not all buffers
commonly employed for liquid chromatography are compatible with mass
spectrometers.
In recent years, there has been a trend to develop ever smaller liquid
chromatography systems. LC systems on micro and even nanoscales have
been demonstrated. Shorter and smaller columns with smaller particles offer
faster analysis times, decreased solvent consumption and require less


22
Chromatography Course Dr Ehab Aboueladab-Lecturer of Biochemistry-Mansoura University-Branch Damietta


sample. The differences between preparative, analytical, micro and nano
LC are summarized in Table 1.2.

Table 1.2. Differences between preparative, analytical, micro and nano
liquid chromatography.
















23
Chromatography Course Dr Ehab Aboueladab-Lecturer of Biochemistry-Mansoura University-Branch Damietta




Chapter Two
ADSORPTION CHROMATOGRAPHY
In adsorption chromatography the compounds to be separated are
adsorbed onto the surface of a solid material. The compounds are desorbed
from the solid adsorbent by eluting solvent.
1-Separation of the compounds depends on
1-The relative balance between the affinities of the compounds
for the adsorbent and their solubility in the solvent.
2-The chemical nature of the substances.
3-The nature of the solvent.
4-The nature of the adsorbent.
Solid adsorbents commonly used are alumina, silica gel, charcoal
(active carbon), cellulose, starch, and sucrose.
Solvents commonly used are hexane, benzene, petroleum ether, diethyl
ether, chloroform, methylene chloride, various alcohols (ethyl, propyl,
n-buryl and t-butyl alcohols), and various aqueous buffers and salts,
some in combination with organic solvents
Adsorption chromatography is a column that is packed with the adsorbents.
The adsorbent is prepared and poured into the column with an inert support
at the bottom. Suitable supports include plastic discs, or sheets of nylon
or Teflon fabrics.
The adsorbent bed must be homogeneous and free of bubbles, cracks,
or spaces between the adsorbents and the walls of the column.


24
Chromatography Course Dr Ehab Aboueladab-Lecturer of Biochemistry-Mansoura University-Branch Damietta


The choice of the eluting solvent, although very important, depends on
the nature of the substances to be separated and the adsorbent, and
hence affords considerable latitude. The process of eluting the sample
components from the adsorbent by the solvent is termed development. As
illustrated in Figure 2-1, the compounds in the mixture that are more
soluble in the solvent and have less affinity for the adsorbent move
more quickly down the column.
If the substances are colored, as they were in Tswett's experiment, they are
readily visible as they separate, However, many substances are not colored,
and in these instances, as the development proceeds, fractions are collected
at the bottom of the column, and the different fractions are analyzed for
compounds of the types that are being separated, For example, if proteins
are being separated, the fractions would be analyzed for protein by
measurement of the UV absorbance at 280 nm. If carbohydrates or
nucleic acids are being separated analytical measurements for
carbohydrates or nucleic acids. The collection of fractions by an automatic
fraction collector.


25
Chromatography Course Dr Ehab Aboueladab-Lecturer of Biochemistry-Mansoura University-Branch Damietta



Figure 2-1: Collection of fractions from a column by an automatic fraction

a device that accumulates from an elution column the same predetermined
volume in each of a series of tubes that automatically change position when
the proper volume has been collected .This may be accomplished in various
ways. For example, set volume, with a timer, or by counting drops with a
drop counter. The latter is frequently used and is usually the most reliable
and flexible. The fraction collector may be Equipped with a detection cell
that automatically measures some parameter of the solution going into the
tubes and may correlated with fraction number and automatically recorded.
The detection cell is frequently a small spectrophotometer that can measure
absorbances at a fixed wavelength or at variable wavelengths. Other
detecting cell use index of refraction, optical rotation, and other properties.


26
Chromatography Course Dr Ehab Aboueladab-Lecturer of Biochemistry-Mansoura University-Branch Damietta



Figure 2-2: Adsorption chrornatography
A = adsorbent, S=Sample, ES = eluting solvent
(A) Application of sample to the column.
(B) Adsorption of sample onto adsorbent.
(C)Addition of elution solvent.
(D) and (E) Partial fraction of sample components.
(F) Complete fractionation of sample. (G) and (H) Separation of all three
components at various stages on the adsorbents.
(I) Elution of the first component from the column.

The substances adsorbed on the column support can be eluted in three ways
In the simplest method, a single solvent continuously flows through
the column until the compounds have been separated and eluted from
the column
Stepwise elution, in which two or more different solvents of fixed
volume are added in sequence to elute the desired compounds.
Gradient elution, in which the composition of the solvent is
continuously changing. The latter method is used to effect separations
that are difficult because of a tendency of component to be eluted in
broad. Trailing bands when a single solvent is used. Gradient elution


27
Chromatography Course Dr Ehab Aboueladab-Lecturer of Biochemistry-Mansoura University-Branch Damietta


frequently provides a means of sharpening the bands, a simple linear
gradient has two solvents, A and B, in which A is the starting solvent
and B is the final solvent. Solvent B is allowed to flow into solvent A as
solvent A flows into the column. The composition of solvent A is, thus,
constantly changing as solvent B is constantly being added to A (Fig.
2-3).
Gradients other than linear gradients (e.g., exponential, concave. or
convex) may be obtained by introducing a third vessel and varying the
composition of the solvents in the vessels. These eluting methods are also
used with other column chromatographic methods.
2-Activation of adsorbent
Many adsorbents such as alumina, silica gel, and active carbon and
Mg silicate can obtain commercially, but they require activation before use.
Activation is achieved by heating and there is usually an optimum
temperature for activation, for e.g. alumina is about 400
o
C. For reduced
activity by the controlled addition of water, and the subsequent activity is

Figure 2-3: Gradient elution. Flow of solvent B into solvent A With mixing,
continuously changing the composition of solvent A as it flows into column


28
Chromatography Course Dr Ehab Aboueladab-Lecturer of Biochemistry-Mansoura University-Branch Damietta


related to the amount of water added. Brookman and Schodder
established five grades of alumina Grade I is the most active and the is
simply alumina heated at about 350
0
C for several hours. Grade II has about
2-3% water, Grade III 5-7%, Grade IV 9-11 %, Grade V film. (Least active)
about 15%.
3-Retention
The retention is a measure of the speed at which a substance moves in a
chromatographic system. In continuous development systems like HPLC or
GC, where the compounds are eluted with the eluent, the retention is usually
measured as the retention time Rt or tR, the time between injection and
detection. In interrupted development systems like TLC the retention is
measured as the retention factor R
f
, the run length of the compound divided
by the run length of the eluent front:

The retention of a compound often differs considerably between
experiments and laboratories due to variations of the eluent, the stationary
phase, temperature, and the setup. It is therefore important to compare the
retention of the test compound to that of one or more standard compounds
under absolutely identical Conditions.
4-Plate theory
The plate theory of chromatography was developed by Martin and
Synge. The plate theory describes the chromatography system, the mobile
and stationary phases, as being in equilibrium. The partition coefficient K
is based on this equilibrium, and is defined by the following equation:


29
Chromatography Course Dr Ehab Aboueladab-Lecturer of Biochemistry-Mansoura University-Branch Damietta



K is assumed to be independent of concentration, and can change if
experimental conditions are changed, for example temperature is increased
or decreased. As K increases, it takes longer for solutes to separate. For a
column of fixed length and flow, the retention time (tR) and retention
volume (Vr) can be measured and used to calculate K
5- Column chromatography
1. Small plug of wool (or cotton)
2. Sand to cover "dead volume"
3. Silica gel, length = 5.5 - 6 inch (Note 1inch=2.54cm).
4. Tap column on bech (carefully) to remove air bubbles inside
the column
5. Add solvent system
6. Add sand on top of silica
7. The top of the silica gel should not be allowed to run dry.
8. Sample is diluted (20-25% solution)
9. The sample is applied by pipette
10. Solvent used to pack the column is reused
11. Walls of column are washed with a few milliliters of eluant
12. Column is filled with eluant
13. Flow controller is secured to column and adjusted 2.0 in / min.


30
Chromatography Course Dr Ehab Aboueladab-Lecturer of Biochemistry-Mansoura University-Branch Damietta





31
Chromatography Course Dr Ehab Aboueladab-Lecturer of Biochemistry-Mansoura University-Branch Damietta



Figure 2-5


32
Chromatography Course Dr Ehab Aboueladab-Lecturer of Biochemistry-Mansoura University-Branch Damietta



Table 2-1: Common adsorbents and the type of compounds
Solid Suitable for separation of
Alumina Steriods, vitamins, ester, and alkaloids
Silica gel Steriods, amino acids, alkaloids
Carbon Peptides, carbohydrates, amino acid
Magnesium carbonate Porphyrins
Magnesium silicate Steriods, ester, glycerides, alkaloids
Magnesia Similar to alumina.
Ca(OH)
2
Carotenoids.
CaCO
3
Carotenoids and xanthophylls.
Ca Phosphate Enzymes, protein, and polynucleotide
Starch Enzymes.
Sugar Chlorophyll.











33
Chromatography Course Dr Ehab Aboueladab-Lecturer of Biochemistry-Mansoura University-Branch Damietta



Chapter Three
PAPER CHROMATOGRAPHY
Paper chromatography is a type of liquid-liquid partition
chromatography that may be performed by ascending or descending
solvent flow. Each mode has its advantages and disadvantages.
Ascending chromatography involves relatively simple and inexpensive
equipment compared with descending chromatography and usually gives
more uniform migration with less diffusion of the sample "spots."
Descending chromatography, on the other hand is usually faster because
gravity aids the solvent flow and with substances of relatively low mobility.
The solvent can run off the paper. Giving a longer path for migration. To
resolve compounds with low mobility. Ascending chromatography may be
performed more than once utilizing a multiple-ascent technique.
For descending chromatography, papers 22 cm wide and 56 cm
long can be used. To facilitate the flow of solvent from the paper, the bottom
of the paper is serrated with a pair of pinking shears. Three pencil lines are
drawn 25 mm apart at the top of the sheet, and small aliquot of the sample
(10-50 ml) is placed at a marked spot on the third line. The spot is kept as
small as possible by adding the aliquot in small increments. With drying in
between. This may be expedited with a hair dryer. Several samples,
including standards, are placed 15-25 mm apart.
The paper is then folded along the other two lines and placed in the
solvent trough of the descending tank (Fig. 3-1). This has been equilibrated
with solvent beforehand to ensure a saturated atmosphere. The paper is
irrigated with solvent until the solvent reaches the bottom or for a longer


34
Chromatography Course Dr Ehab Aboueladab-Lecturer of Biochemistry-Mansoura University-Branch Damietta


period, allowing the solvent to flow off the end of the paper, if necessary. The
chromatogram is then removed dried and developed to reveal the locations
of the compounds. (Part II gives methods of locating carbohydrates, amino
acids. proteins. nucleotides and nucleic acids and lipids.)
In ascending chromatography, a paper approximately 25 cm x 25
cm is used. A pencil line 20-25 mm from the bottom is drawn across the
paper

Fig. 3-1 Steps in descending paper chromatography

and aliquots (10-50l) of the samples and standards are spotted
approximately 15-25 mm apart along the line. The spots are dried and the
paper is rolled into a cylinder and stapled so that the ends of the paper are
not touching (Fig. 3-2). Solvent is poured into the bottom of a
chromatographic chamber, and the cylinder is placed inside. The chamber is
closed and solvent is allowed to flow up


35
Chromatography Course Dr Ehab Aboueladab-Lecturer of Biochemistry-Mansoura University-Branch Damietta



Fig.3-2 Steps in ascending paper chromatography

The paper by capillary action. The chamber may be a simple wide-mouth,
screw top, gallon jar or a cylinder with a ground-glass edge and a glass plate
top. As with descending chromatography, the chamber should be
equilibrated with solvent beforehand. Contrary to a popular misconception, if
the chamber has been sealed and is airtight, the paper does not have to be
removed as soon as the solvent reaches the top. When multiple ascents are
performed, the paper is removed, thoroughly dried, and returned to the
chamber for another ascent of solvent.
The resolved compounds on a paper chromatogram may be detected by
their color if they are colored, by their fluorescence if they are fluorescent, by
a color that is produced from a chemical reaction on the paper after spraying
or dipping the chromatogram with various reagents, or by autoradiography if
the compounds are radioactive. Identification of compounds on a
chromatogram is usually based on a comparison with authentic compounds


36
Chromatography Course Dr Ehab Aboueladab-Lecturer of Biochemistry-Mansoura University-Branch Damietta


(standards). A quantitative comparison may be made by measuring the R
f
,
which is the ratio of the distance the compound migrates to the distance the
solvent migrates. A better comparison is the ratio of the distance a particular
compound migrates to the distance a particular standard migrates. For
example, in the separation of carbohydrates, the standard might be glucose
and the ratio would be R
Glc
or for amino acids, the standard might be glycine
and the ratio would be R
Gly

A useful modification is two-dimensional paper chromatography, in
which the sample is spotted in the lower left-hand corner and irrigated in the
first dimension with solvent A. The chromatogram is removed from the
solvent dried, turned 90, and irrigated in the second dimension with solvent
B, giving a two-

Fig. 3-3 Two-dimensional paper or thin-layer chromatography


37
Chromatography Course Dr Ehab Aboueladab-Lecturer of Biochemistry-Mansoura University-Branch Damietta



dimensional separation (Fig. 3-3). An application of this procedure has
been developed for the study of enzyme specificity in which a solution of the
enzyme is sprayed onto the chromatogram between the first irrigation and
the second to see what products are formed by the action of the enzyme on
the compounds separated in the first dimension.


Fig.3-4. Elution of compounds from paper chromatograms for preparative
chromatography or quantitative determination

eluted with water. To accomplish the elution, tabs of chromatographic paper
are stapled to the narrow ends of each strip. As shown in Figure 3.4, one
end is fitted with two pieces of glass (cut microscope slides), which arc held


38
Chromatography Course Dr Ehab Aboueladab-Lecturer of Biochemistry-Mansoura University-Branch Damietta


together with rubber bands, and the bottom end is cut tapered, like a pipet
tip. This assembly is played so that one end lies in a chromatographic trough
containing water, and the elution of the strip occurs by capillary flow of the
water down the paper strip into a baker.
Usually less than 1 mL of water is sufficient to effect quantitative
elution, the samples are quantitatively diluted to a specific volume, and a
chemical analysis is performed for the specific compound separated. This
technique also may be used as a preparative procedure to obtain small
quantities of pure compound from a mixture of compounds.
In an alternate quantitative procedure, the compounds in the sample are
radioactively labeled and separated in the usual way, and an autoradiogram
is prepared. The labeled compounds are located on the chromatogram by
comparing their positions on the autoradiogram. The radioactive compounds
are cut out and placed into a liquid scintillation cocktail, and the radioactivity
is determined by heterogeneous liquid scintillation counting

In paper chromatography, the mobile phase (solvent) carries the
components of the sample on the stationary phase (filter paper) separating
them according to the differences in the migration rate (depends on the
molecular weight , polarity and adsorption ability)
Components
For one-dimensional paper chromatography, either ascending or
descending development can be carried out in simple units. Descending
development is more often used because it is faster and more suitable for
long paper sheets.
The stationary phase (filter paper)


39
Chromatography Course Dr Ehab Aboueladab-Lecturer of Biochemistry-Mansoura University-Branch Damietta


The mobile phase (solvent may be in a reservoir)
Procedures
1. Make the initial line on the paper.
2. Apply the solvent alone on the initial line.
3. Wait till the solvent migration is stopped, then make the final line.
4. Spot the sample, and then apply the solvent either in ascending or
descending or concentric manner.
5. In case of colored sample: Calculate the rate of flow (Rf) directly then
compare it with stander in order to know the unknown sample (qualitatively).
6. In case of the colorless sample: use UV-lamb to detect the spot
position then determine the (Rf). Rf depends on the temp., solvent, type of
paper Rf = distance of sample migration / distance of solvent migration
Applications
1. Separation of amino acids
2. Separation of the plant pigments
Advantages
1. Simple
2. Cheap
Disadvantages
1. Time consuming.
2. Need high quantity of sample.
3. With weak solvent power.
4. limited use
5. Difficulty detection of spots
6. Difficulty isolation of separated substances.



40
Chromatography Course Dr Ehab Aboueladab-Lecturer of Biochemistry-Mansoura University-Branch Damietta



Chapter Four
Thin layer chromatography
This technique is particularly useful for the separation of very small
amounts of material. The general principle involved is similar to that
involved in column chromatography, i.e. it is primarily adsorption
chromatography, although other partition effects may also be involved. A
glass sheet is covered by a uniform thin layer of an adsorbent. Adsorbents
used in TLC, differ from column adsorbents. It contains a binding agent
such as calcium sulphate, which facilitates the adsorbent sticking to the
glass plate. The plates are prepared by spreading slurry of adsorbent in
water over them, starting at one end, and moving progressively to the other
and then drying them in an oven at 100-120C. Drying serves to remove
the water and to leave a coating of adsorbent on the plate. Equipment is
available which will ensure the production of an even coating of adsorbent
over a series of glass plates. The normal thickness of slurry layer used is
0.25 mm for qualitative analysis, but layers up to 5-10 mm thick may be
made for preparative work.



41
Chromatography Course Dr Ehab Aboueladab-Lecturer of Biochemistry-Mansoura University-Branch Damietta


The sample is applied to the plate by micropipette or syringes, as spot 2.5
cm from one end and at least an equal distance from the edge. The solvent
is removed from the sample by the use of an air blower. All spots should be
placed on equal distance from the end of the plate.
Separation takes place in glass tank which contains the developing solvent
(mobile phase) to a depth of 1.5 cm , this is allowed to stand for at least 1
hour with a glass plate over the top of the tank to ensure that the
atmosphere within the tank becomes saturated with solvent vapor.
Then, the thin layer plate is placed vertically in the tank so that, it stands in
the solvent with the end bearing the sample in the solvent.
The cover plate is replaced and separation of the compounds then occurs as
the solvent travels up the plate. After the solvent had reached the wanted
level, the run is stopped. The chromatographic separation is completed the
spots of the component substances can be detected by different
methods:
1-Many commercially available TLC adsorbents contain a fluorescent
dye, the plate is examined under UV light, the separated components
show up as blue, green, black area.
2. Spraying the plate with 50% sulphuric acid and heating so, the
compounds become charred and show spots
3. Spraying the plates with specific color reagents will stain up certain
compounds e.g. ninhydrin for amino acid (aa) , aniline for aldoses.
Solvents
Universal TLC System:
Petroleum ether - ethyl acetate
Very polar solvent additives:


42
Chromatography Course Dr Ehab Aboueladab-Lecturer of Biochemistry-Mansoura University-Branch Damietta


Methanol > ethanol > isopropanol

Moderately polar additives:
Acetonitrile > ethyl acetate > chloroform, dichloromethane > diethyl ether >
toluene
Non-polar solvents:
Cyclohexane, petroleum ether, hexane, pentane
TLC Visualization (Detecting the spots)
Non-destructive techniques:
1. Ultraviolet lamp. Shows any UV-active spots
2. Plate can be stained with iodine.
Bottle containing silica and a few crystals of iodine (especially good for
unsaturated compounds)
Destructive techniques
Staining Solutions immerse the plate as completely as possible in the stain
and remove it quickly. Heat carefully with a heating
Stains Use/Comments
Anisaldehyde Good general reagent, gives a range of colors
PMA Good general reagent, gives blue/green spots
Vanillin Good general reagent, gives a range of colors
Ceric sulfate Fairly general reagent, gives a range of colors
DNP Mainly for aldehydes and ketones, gives orange
spots
Permangante Mainly for unsaturated compounds and alcohols,
gives yellow spots


43
Chromatography Course Dr Ehab Aboueladab-Lecturer of Biochemistry-Mansoura University-Branch Damietta





Thin-Layer Chromatography of Amino acids
Amino acids may be separated by two-dimensional TLC using either
silica gel or cellulose as the separating medium. Two different solvents are
used for each type of TLC plate and a different type of separation is
achieved for each type. The amino acids are visualized with two types of
ninhydrin spray for the silica gel and the cellulose gel media.
Ninhydrin Sprays for amino acid detection
For silica gel TLC: The plate is sprayed with a solution of 300 mg of
ninhydrin + 3 ml of glacial acetic acid + 100 ml of butyl alcohol and heated
for 10 minutes at 110C.
For cellulose TLC:
The plate is sprayed with a solution of 500 mg of ninhydrin + 350 ml of
absolute ethanol + 100 ml of glacial acetic acid + 15 ml of 2,4,6-
trimethylpyridine and heated for 10 minutes at 110C.
Two-dimensional TLC separation of amino acids.
On silica gel G with
Solvent I, chlorolorm-17% methanol (v/v)-ammonia (2:2:1, v/v/v) and
Solvent II, phenol-water (75:25, v/v).


44
Chromatography Course Dr Ehab Aboueladab-Lecturer of Biochemistry-Mansoura University-Branch Damietta




on cellulose MN 300 with
Solvent III, 1-butanol-acetone-diethylamine-water (10:10:2:5,v/v/v/v, pH
12.0) and
Solvent IV, 2-propanol-formic acid (99%)-water (40:2:10, v/v/v, pH 2.5)


Thin-Layer Chromatography of Carbohydrates


45
Chromatography Course Dr Ehab Aboueladab-Lecturer of Biochemistry-Mansoura University-Branch Damietta


Carbohydrates may be separated on commercial silica gel plates using
a variety of solvents to achieve specific separations. The results of the
separation depend on the particular plate used. Whatman K5 silica gel and
Merck silica gel 60 plates give good results.
Solvent for TLC separations of carbohydrates
Solvent: Acetonitrile-water (35:15, v/v) with four ascents (45 minutes each
for a 20-cm plate) will separate mono-, di, and trisaccharides
The visualization of carbohydrates on thin layer silica gel plates is
obtained by spraying with sulfuric acid-methanol (1: 3, v/v) followed by
heating for 10 minutes at 110-120C. Most carbohydrates give black to
brown spots on a white background.

Examples of some TLC separation systems
Compounds Adsorbent Solvent system (v/v)
Amino acids Silica Gel G 96% Ethanol/water (70/30)
Butan-1-ol/acetic acids/
water (80/20/20)
Mono and di
saccharides
Kieselguhr G (sodium
acetate)
Kieselguhr G
(sodium phosphate
pH5)
Ethyl acetate/propan-1-ol
(65/35). Butan-1-ol /
acetone/phosphate buffer
pH5 (40/50/10)
Neutral lipids Silica Gel G Petroleum ether/diethyl
ether/acetone (90/10/1)


46
Chromatography Course Dr Ehab Aboueladab-Lecturer of Biochemistry-Mansoura University-Branch Damietta


Cholesterol
Esters
Silica Gel G Carbon tetrachloride/
chloroform (95/5)
Carotenoids Kieselguhr G Petroleum ether/propan-1-
ol (99/1)
Phospholipids Silica Gel G Chloroform/methanol/water
(65/25/4)

Advantages of TLC.
The speed at which separation is achieved. With a volatile solvent as
the mobile phase the time involved may be as low as 30 minutes, but even
with non-volatile solvents the time involved is rarely longer than 90
minutes.
Summary for TLC
Principle
As in paper chromatography
Components
glass or plastic plate: as a support to the stationary phase
stationary phase (silica gel, alumina or agar)
mobile phase solvent system
Procedures
(a) Preparing the plate
1- Prepare a glass plate. for example (20X20)Cm
2- Dissolve suitable amount of the silica gel in water path.
3- Spread it on glass plate homogeneously. Then wait till solidification.
(b) Running the sample


47
Chromatography Course Dr Ehab Aboueladab-Lecturer of Biochemistry-Mansoura University-Branch Damietta


1- Make the initial and final line as in PC
2- Spot the sample on the initial line and then apply the solvent either in
ascending or descending or concentric manner
3- For example: add the plate on a tank containing the solvent
(ascending) where the solvent move by the capillary action carrying
with it the components of the sample.
4- The plate is removed from the tank and dried. (Additional separation
can be achieved in two dimensional TLC)
5- According to the sample type (if colored, if colorless, if florescent),
identification (qualitative) occurs either by Rf, UV, spraying colored
reagent or autoradiography.
6- Compare it with stander in order to know the unknown sample
components.
7- We can separate the sample by cutting the silica layer by spatula, then
dissolve it with the same solvent then filter for further purification.
Applications
1- Environmental application from water analysis (especially pesticides) to
plant residues
2- Pharmaceutical applications from stability and impurity studies to drug
monitoring in biological fluid
3- Biomedical compounds (organic acids, lipids, carbohydrates and
steroids)
4- Food analysis from carcinogens, drug residues , preservatives and
flavors
Disadvantages
1- Time consuming


48
Chromatography Course Dr Ehab Aboueladab-Lecturer of Biochemistry-Mansoura University-Branch Damietta


2- Limited to non-volatile compounds
3- Less accurate and less sensitive

Advantages
1- Need small quantity of sample.
2- With greater solvent power.
3- easy detection of spots
4- easy isolation of separated substances










49
Chromatography Course Dr Ehab Aboueladab-Lecturer of Biochemistry-Mansoura University-Branch Damietta




The procedure of two-dimensional thin-layer chromatography




50
Chromatography Course Dr Ehab Aboueladab-Lecturer of Biochemistry-Mansoura University-Branch Damietta



Developing solvent mixtures that have been recommended for two dimensional TLC separation of
underivatised amino-acids


Organic component of the solvent continues migrating, thus forming the mobile


51
Chromatography Course Dr Ehab Aboueladab-Lecturer of Biochemistry-Mansoura University-Branch Damietta


phase. Therefore, compounds soluble to organic component move faster than
compounds soluble to aqueous component. -Thus, molecules are separated
according to their polarities.








52
Chromatography Course Dr Ehab Aboueladab-Lecturer of Biochemistry-Mansoura University-Branch Damietta





Chapter Five
Gel filtration
Biomolecules are purified using chromatography techniques that separate
them according to differences in their specific properties, as shown in Figure
5.1. and Table 5.1.
Property Technique
Size Gel filtration (GF), also called size
exclusion
Charge Ion exchange chromatography (IEX)
Hydrophobicity Hydrophobic interaction
chromatography (HIC)
Reversed phase chromatography
(RPC)
Biorecognition (ligand
specificity)
Affinity chromatography (AC)
Table 5.1.



53
Chromatography Course Dr Ehab Aboueladab-Lecturer of Biochemistry-Mansoura University-Branch Damietta


Fig. 5.1 Separation principles in chromatography purification.

Gel filtration has played a key role in the purification of enzymes,
polysaccharides, nucleic acids, proteins and other biological
macromolecules. Gel filtration is the simplest and mildest of all the
chromatography techniques and separates molecules on the basis of
differences in size. The technique can be applied in two distinct ways:
1. Group separations:
The components of a sample are separated into two major groups
according to size range. A group separation can be used to remove high or
low molecular weight contaminants (such as phenol red from culture fluids)
or to desalt and exchange buffers.



54
Chromatography Course Dr Ehab Aboueladab-Lecturer of Biochemistry-Mansoura University-Branch Damietta



2. High resolution fractionation of biomolecules:
The components of a sample are separated according to differences
in their molecular size. High resolution fractionation can be used to isolate
one or more components, to separate monomers from aggregates, to
determine molecular weight or to perform a molecular weight distribution
analysis.
Gel filtration can also be used to facilitate the refolding of denatured proteins
by careful control of changing buffer conditions.
Gel filtration is a robust technique that is well suited to handling biomolecules
that are sensitive to changes in pH, concentration of metal ions or co-
factors and harsh environmental conditions. Separations can be performed
in the presence of essential ions or cofactors, detergents, urea,
guanidine hydrochloride, at high or low ionic strength, at 37 C or in the
cold room according to the requirements of the experiment

Gel filtration in practice
Gel filtration separates molecules according to differences in size as
they pass through a gel filtration medium packed in a column. Unlike ion
exchange or affinity chromatography, molecules do not bind to the
chromatography medium so buffer composition does not directly affect
resolution (the degree of separation between peaks).
Separation by gel filtration
Gel filtration medium is packed into a column to form a packed bed. The
medium is a porous matrix in the form of spherical particles that have been
chosen for their chemical and physical stability, and inertness (lack of


55
Chromatography Course Dr Ehab Aboueladab-Lecturer of Biochemistry-Mansoura University-Branch Damietta


reactivity and adsorptive properties). The packed bed is equilibrated with
buffer which fills the pores of the matrix and the space in between the
particles. The liquid inside the pores is sometimes referred to as the
stationary phase and this liquid is in equilibrium with the liquid outside the
particles, referred to as the mobile phase as shown in Figure 2.
Gel filtration is used in group separation mode to remove small
molecules from a group of larger molecules and as a fast, simple solution for
buffer exchange. Small molecules such as excess salt (desalting) or free
labels are easily separated. Samples can be prepared for storage or for
other chromatography techniques and assays. Gel filtration in group
separation mode is often used in protein purification schemes for
desalting and buffer exchange


56
Chromatography Course Dr Ehab Aboueladab-Lecturer of Biochemistry-Mansoura University-Branch Damietta





57
Chromatography Course Dr Ehab Aboueladab-Lecturer of Biochemistry-Mansoura University-Branch Damietta




Fig. 5. 2. Common terms in gel filtration
Sephadex G-10, G-25 and G-50 are used for group separations. Large
sample volumes up to 30% of the total column volume (packed bed) can be
applied at high flow rates using broad, short columns. Figure 3 shows the
elution profile (chromatogram) of a typical group separation. Large
molecules are eluted in or just after the void volume, Vo as they pass
through the column at the same speed as the flow of buffer. For a well
packed column the void volume is equivalent to approximately 30% of
the total column volume. Small molecules such as salts that have full
access to the pores move down the column, but do not separate from each
other. These molecules usually elute just before one total column volume,
Vt, of buffer has passed through the column. In this case the proteins are


58
Chromatography Course Dr Ehab Aboueladab-Lecturer of Biochemistry-Mansoura University-Branch Damietta


detected by monitoring their UV absorbance, usually at A280 nm, and the
salts are detected by monitoring the conductivity of the buffer.


Fig. 5.3. Typical chromatogram of a group separation. The UV (protein) and
conductivity (salt) traces enable pooling of the desalted fractions and
facilitate optimization of the separation.

The theoretical elution profile (chromatogram) of a high resolution
fractionation. Molecules that do not enter the matrix are eluted in the void
volume, Vo as they pass directly through the column at the same speed as
the flow of buffer. For a well packed column the void volume is equivalent to
approximately 30% of the total column volume (packed bed). Molecules with
partial access to the pores of the matrix elute from
the column in order of decreasing size. Small molecules such as salts that
have full access to the pores move down the column, but do not separate
from each other. These molecules usually elute just before one total column

Sample: (His)6 protein eluted from HiTrap
Chelating HP with
sodium phosphate 20 mM,
sodium chloride 0.5 M,
Imidazole 0.5 M, pH 7.
Column: HiTrap Desalting 5 ml
Buffer: Sodium phosphate 20 mM,
Sodium chloride 0.15 M, pH 7.0
Void volume :Vo,
Total column volume :Vt


59
Chromatography Course Dr Ehab Aboueladab-Lecturer of Biochemistry-Mansoura University-Branch Damietta


volume, Vt, of buffer has passed through the column, Fig. 5.4.

Fig. 5.4.Theoretical chromatogram of a high resolution fractionation (UV
absorbance).


60
Chromatography Course Dr Ehab Aboueladab-Lecturer of Biochemistry-Mansoura University-Branch Damietta


Separation examples

Fig. 5.5 Cytochrome C, Aprotinin, Gastrin I, Substance P,
(Gly)
6
, (Gly)
3
and Gly

Media Selection
Chromatography media for gel filtration are made from porous
matrices chosen for their inertness and chemical and physical stability. The
size of the pores within a particle and the particle size distribution are
carefully controlled to produce a variety of media with different selectivities.
Today's gel filtration media cover a molecular weight range from 100 to
80 000 000, from peptides to very large proteins and protein complexes.
Figure.5.7.


61
Chromatography Course Dr Ehab Aboueladab-Lecturer of Biochemistry-Mansoura University-Branch Damietta



Sephacryl is suitable for fast, high recovery separations at laboratory and
industrial scale


62
Chromatography Course Dr Ehab Aboueladab-Lecturer of Biochemistry-Mansoura University-Branch Damietta


Sephadex is ideal for rapid group separations such as desalting and buffer
exchange.
Sephadex is used at laboratory and production scale, before, between or
after other chromatography purification steps.
Determination molecular weight
V
e
V
0

K
av
= --------------
V
t
V
0


Where V
e
= elution volume for the protein
V
o
= column void volume
V
t
= total bed volume
On semi logarithmic graph paper, plot the K
av
value for each protein
standard (on the linear scale) against the corresponding molecular
weight (on the logarithmic scale). Draw the straight line which best fits the
points on the graph. Then, calculate the corresponding K
av
for the
component of interest and determine its molecular weight from the
calibration curve.
Sephadex:
Rapid group separation of high and low molecular weight substances,
such as desalting, buffer exchange and sample clean up
Sephadex is prepared by cross-linking dextran with epichlorohydrin.
Variations in the degree of cross linking create the different Sephadex media
and influence their degree of swelling and their selectivity for specific
molecular sizes (Table.5.2).


63
Chromatography Course Dr Ehab Aboueladab-Lecturer of Biochemistry-Mansoura University-Branch Damietta



Product Fractionation
range, Mr
(globular
proteins)
pH stability Bed
volume
ml/g
dry
Sephad
ex
Particle size,
wet
Sephadex G-
10
<710
2
Long term: 213
Short term: 213
2-3 55165 m
Sephadex G-
25 Coarse
110
3
510
3
Long term: 213
Short term: 213
4-6 170520 m
Sephadex G-
25 Medium
110
3
510
3
Long term: 213
Short term: 213
4-6 85260 m
Sephadex G-
25 Fine
110
3
510
3
Long term: 213
Short term: 213
4-6 35140 m
Sephadex G-
25 Superfine
110
3
510
3
Long term: 213
Short term: 213
4-6 1770 m
Sephadex G-
50 Fine
110
3
310
4
Long term: 210
Short term: 213
9-11 40160 m

Sephadex G-10 is well suited for the separation of biomolecules such as
peptides (Mr >700) from smaller molecules (Mr <100).
Sephadex G-50 is suitable for the separation of molecules Mr >30000 from
molecules Mr<1500 such as labeled protein or DNA from unconjugated
dyes. The medium is often used to remove small nucleotides from longer
chain nucleic acids.


64
Chromatography Course Dr Ehab Aboueladab-Lecturer of Biochemistry-Mansoura University-Branch Damietta


Sephadex G-25 is recommended for the majority of group separations
involving globular proteins. These media are excellent for removing salt
and other small contaminants away from molecules that are greater than
Mr 5000. Using different particle sizes enables columns to be packed
according to application requirements
Sephadex is prepared by cross-linking dextran with epichlorohydrin,
illustrated in Figure 5.10. The different types of Sephadex vary in their
degree of cross-linking and hence in their degree of swelling and selectivity
for specific molecular sizes, as shown

Fig. 5.10. Partial structure of Sephadex.
Why use different techniques at each stage
In order to final removal of trace contaminants. Adjustment of pH, salts
or additives for storage. Then, end product of required high level purity


65
Chromatography Course Dr Ehab Aboueladab-Lecturer of Biochemistry-Mansoura University-Branch Damietta


Therefore, The technique chosen must discriminate between the target
protein and any remaining contaminants

Gel Filtration = Gel Permeation Chromatography =
Size Exclusion Chromatography
Size exclusion chromatography (SEC), also called gel permeation
Chromatography (GPC) or gel filtration chromatography (GFC) is a
technique for separates molecules according to their molecular size. Gel
particles form the stationary phase of this type of chromatography; the
mobile phase is the solution of molecules to be separated and the eluting
solvent, which most frequently is water or a dilute buffer. The sample is
applied to the gel, if the molecules are too large for the pores; they never
enter the gel and move outside the gel bed with the eluting solvent. Thus,
the very large molecules in a mixture move the fastest through the gel bed
and the smaller molecules, which can enter the gel pores, are retarded and
move more slowly through the gel bed. In gel chromatography, molecules
are, therefore, eluted in order of decreasing molecular size






66
Chromatography Course Dr Ehab Aboueladab-Lecturer of Biochemistry-Mansoura University-Branch Damietta






Fig.5.11 Gel permeation chromatography. Open circles represent porous gel
molecules: large solid Circles represent molecules too large to enter the gel
through the pores, and smaller solid circles represent molecules capable of
entering the gel pores

Three types of polymers are principally used-dextran, polyacrylamide,
and agarose
Dextran is a polysaccharide composed of (-1--->6)-linked glucose residues
with (-1, 3) branch linkages. It is synthesized from sucrose by an enzyme


67
Chromatography Course Dr Ehab Aboueladab-Lecturer of Biochemistry-Mansoura University-Branch Damietta


produced by the bacterium Leuconostoc mesenteroides B-512F. The
dextran is cross-linked to various extents by reaction with epichlorohydrin
to give gel beads with different pore sizes Fig 5.12. Cross-linked dextrans
are commercially produced by Pharmacia Fine Chemicals, lnc., (Uppsala,
Sweden), and sold under the trade name Sephadex. Sephadex gels in the
so-called G-series, where the G-numbers refer to the amount of water
gained when the beads are swelled in water (Table 1) have different
degrees of cross-linking, hence different pore sizes. This gives gels that
have capabilities of separating different ranges of molecular weights and
have different molecular exclusion limits. The exclusion limit is the molecular
weight of the smallest peptide or globular protein that will not enter the gel
pore. Sephadex G-10, the highest cross-linked dextran, has a water regain
of about 1mL/g of dry gel and Sephadex G-200, the lowest cross-linked
dextran, has a water regain of about 20 mL/g of dry gel. In the swelling
process, the gels become filled with water.



68
Chromatography Course Dr Ehab Aboueladab-Lecturer of Biochemistry-Mansoura University-Branch Damietta




Fig.5.12. Structure of epichlorohydrin cross linked Dextran









69
Chromatography Course Dr Ehab Aboueladab-Lecturer of Biochemistry-Mansoura University-Branch Damietta


Table 5.1: Properties of gels used in gel permeation (filtration)
chromatography


Gel
Water
regain
(mL/g)
Exclusion
limit
Maximum
hydrostatic
pressure
cm H
2
O
Maximum
flow rate (ml,
min)
Sephadex G-10 1.0 700 200 100
Sephadex G-15 1.5 1500 200 100
Sephadex G-25 2.5 5000 200 50
Sephadex G-50 5.0 30000 200 25
Sephadex G-75 7.5 70000 160 6.4
Sephadex G-100 10.0 150000 96 4.2
Sephadex G-150 15.0 300000 36 1.9
Sephadex G-200 20.0 600000 16 1.0
Sepharose 6B NA 4 x 10
6
200 1.2
Sepharose CL 6B NA 4 x 10
6
>200 2.5
Sepharose 4B NA 20 x 10
6
80 0.96
Sepharose CL 4B NA 20 x 10
6
120 2.17
Sepharose 2B NA 40 x 10
6
40 0.83


70
Chromatography Course Dr Ehab Aboueladab-Lecturer of Biochemistry-Mansoura University-Branch Damietta


Sepharose CL 2B NA 40 x 10
6
50 1.25
Bio-Gel P-2 1.5 1800 >100 110
Bio-Gel P-4 2.4 4000 >100 95
Bio-Gel P-6 3.7 6000 >100 75
Bio-Gel P-10 4.5 20000 >100 75
Bio-Gel P-30 5.7 40000 >100 65
Bio-Gel P-60 7.2 60000 100 30
Bio-Gel P-100 7.5 100000 100 30
Bio-Gel P-150 9.2 150000 100 25
Bio-Gel P-200 14.7 200000 75 11
Bio-Gel P-300 18.0 400000 60 6
Bio-Gel A-0.5m NA 500000 >100 3
Bio-Gel A-1.5m NA 1.5 x 10
6
>100 2.5
Bio-Gel A-5m NA 5 x 10
6
>100 1.5
Bio-Gel A-15m NA 15 x 10
6
90 1.5
Bio-Gel A-50m NA 50 x 10
6
50 1.0
Bio-Gel A-150m NA 150 x 10
6
30 0.5
Bio-Gel is a trade name of Bio-Rad Laboratories, Sephadex and
Sepharose are trade name of Pharmacia Fine Chemical



71
Chromatography Course Dr Ehab Aboueladab-Lecturer of Biochemistry-Mansoura University-Branch Damietta


Polyacrylamide gels are long polymers of acrylamide cross-linked with
N.N'methylene-bisacrylamide (Fig. 5.13).

Fig.5.13. Structure of cross-linked polyacrylamide

The gels are commercially produced by BioRad Laboratories, Richmond.
California, as the Bio-Gel P series. Like the Sephadex G series. the Bio-
Gels differ in degree of cross-linking and in pore size; the Bio-Gels,
however. have a wider range of pore sizes than is available in the Sephadex
G series for the exclusion limits and properties of the different Bio-Gels.
Agarose is a gel material with pore sizes larger than cross-linked dextran or
polyacrylamide. Agarose is the neutral polysaccharide fraction of agar. It is


72
Chromatography Course Dr Ehab Aboueladab-Lecturer of Biochemistry-Mansoura University-Branch Damietta


composed of a linear polymer of D-galactopyranose linked ( 1->4) 3,6
anhydro-L-galactopyranose, which is linked (1-> 3) (Fig.5. 14).

D-galactose (-1->4) 3, 6-Anhydro-L-galactose
Fig.5.14. Structure of the repeating unit of agarose, D-galactopyranose
linked (-1->4) to 3, 6-anhydro-L-galactopyranose, which is linked (-1-3) to
the next D-galactopyranose residue

When the polysaccharide is dissolved in boiling water and cooled, it forms a
gel by forming inter-and intramolecular hydrogen bonds. The pore sizes are
controlled by the concentration of the agarose. High molecular weight
materials such as protein aggregates, chromosomal DNA, ribosomes,
viruses, and cells have been fractionated on agarose gels. Bio-Rad markets
the agarose Bio-Gel A series with different molecular exclusion limits, and
Pharmacia markets agarose as Sepharose and Sepharose CL. The latter
is Sepharose cross-linked by reacting with alkaline 2, 3-dibromopropanol
to give an agarose gel with increased thermal and chemical stability. Table
5.1 gives the properties of the different Sephadex, Bio-Gel, and Sepharose


73
Chromatography Course Dr Ehab Aboueladab-Lecturer of Biochemistry-Mansoura University-Branch Damietta


gels. The separations that may be achieved by gel permeation
chromatography are based on differences in the molecular sizes of the
molecules. The method is used for both preparative and analytical purposes.
The latter has been especially useful in determining the molecular weights of
proteins. The proteins are chromatographed on a gel column and the elution
volume of the protein determined. Proteins with known molecular weights
are also chromatographed and the elution volumes determined. Then, from a
plot of log molecular weight versus elution volume, the molecular weight of
an unknown protein may be determined (Fig. 5.15).

Fig.5.15. Molecular weight determination of proteins by gel permeation chromatography
using Sephadex G-100 as the gel bed: log molecular weight is plotted versus elution
volume.


74
Chromatography Course Dr Ehab Aboueladab-Lecturer of Biochemistry-Mansoura University-Branch Damietta



Gel chromatography provides a rapid and mild method of removing salts and
other small molecules from high molecular weight biomolecules. The sample
containing the biomolecules and the salt is passed over a gel column whose
exclusion limit is below the molecular weight of the biomolecules. The
biomolecules which do not enter the gel emerge in the void volume of the
column, while the salts enter the gel and are retarded, and therefore are
removed from the biomolecules.

















75
Chromatography Course Dr Ehab Aboueladab-Lecturer of Biochemistry-Mansoura University-Branch Damietta


Summary Gel filtration chromatography
(Size-exclusion chromatography)
Principle
This technique separate proteins according to their size and shape, as they
pass through a stationary phase (cross-linked polymer =sephadex) by the
help of mobile phase (without binding). Larger proteins or molecules, which
can not penetrate the sephadex pores, move around the sephadex in space
between them faster than the smaller molecules which may penetrate the
sephadex pores taking long time to elute from the column.
Components
1. Column: as a support to the stationary phase
2. Stationary phase (pours matrix in the form of spherical particles,
stable, inert e.g. sephadex or agarose)
3. Mobile phase (buffer system)
Procedures
1. (Loading step): spherical particles of the sephadex are packed into the
column
2. (Sampling step): sample is applied to the column
3. Buffer (mobile phase) and sample move through the column. The
sample components diffuse in and out of the pores of the matrix (sephadex)
according to their size.
4. Larger proteins or molecules move faster than the smaller molecules
and leave the column first
5. Separation completed as the entire buffer volume is passed.
Applications
1. Separation of neutral proteins and larger molecules including polymers


76
Chromatography Course Dr Ehab Aboueladab-Lecturer of Biochemistry-Mansoura University-Branch Damietta


and biomolecules according to size.
2. The determination of formula weights.

Disadvantages
1. Limited applications
2. Low purification
Advantages
1. Provides a rapid means for separating larger molecules
2. Use only one buffer (coast effective)
3. Do not need elution step because there are no bonds formed.
Note: Gel Filtration
Separation based on size
Molecular sieve chromatography
Size exclusion chromatography
Media composed of crosslinked polymers
Pore size of matrix determines degree of interaction
Larger molecules are excluded and migrate faster
Smaller molecules are included and are retained longer
Dextran (=Sephadex)
Agarose (=Sepharose)
Polyacrylamide choose matrix with desired characteristics
Size range
does not interact with solute
include 0.15-1 M NaCl in buffer
Load sample in smallest possible volume
elute in one column volume


77
Chromatography Course Dr Ehab Aboueladab-Lecturer of Biochemistry-Mansoura University-Branch Damietta


Practical Considerations
Sephadex
Code Range (kDa)
G-25 1-5, G-50 2-30, G-100 4-150, G-150 5-300, G-200 5-600
Applications:
Purification
Desalting
Size determination
Calculating Size




78
Chromatography Course Dr Ehab Aboueladab-Lecturer of Biochemistry-Mansoura University-Branch Damietta


Chapter Six
Ion-exchange chromatography
Ion-exchange chromatography is a variation of adsorption
chromatography in which the solid adsorbent has charged groups chemically
linked to an inert solid. Ions are electrostatically bound to the charged
groups; these ions may be exchanged for ions in an aqueous solution. Ion
exchangers are most frequently used in columns to separate molecules
according to charge. Because charged molecules bind to ion exchangers
reversibly. Molecules can be bound or eluted by changing the ionic strength
or pH of the eluting solvent.
Two types of ion exchanger are available: those with chemically
bound negative charges are called cation exchangers and those with
chemically bound positive charges are called anion exchangers. The
charges on the exchangers are balanced by counterions such as chloride
ions for the anion exchangers and metal ions for the cation exchangers.
Sometimes buffer ions are the counterions. The molecules in solution which
are to be adsorbed on the exchangers also have net charges which are
balanced by counterions. As an example of an ion-exchange process, let us
say that the molecules to he adsorbed from solution have a negative charge
(X
-
), which is counterbalanced by sodium ions (Na
+
). Such negatively


79
Chromatography Course Dr Ehab Aboueladab-Lecturer of Biochemistry-Mansoura University-Branch Damietta


charged molecules can be chromatographed on an anion exchanger
(A
+
), which has chloride ions as the counterion to give A
+
Cl
-
. When (Na
+
X
-
)
molecules in solution interact with the ion exchanger, the X
-

displaces the
chloride ion from the exchanger and becomes electrostatically bound to give
A
+
X
-
, simultaneously releasing sodium ions. This process of ion exchange is
illustrated in Figure 1. A similar but opposite process would take place for
positively charged molecules (Y
+
Cl
-
) which would be chromatographed on
cation exchangers (C
-
Na
+
). Thus the cation exchangers will bind positively
charged molecules from solution and the anion exchangers will bind
negatively charged molecules from solution.
One of the inert materials used in this type of chromatography is
cross-linked polystyrene, to which the charged groups are chemically
bound. In the separation of biologically important macromolecules,
such as enzymes and proteins.

Figure 6.1. The process of anion-exchange chromatography


80
Chromatography Course Dr Ehab Aboueladab-Lecturer of Biochemistry-Mansoura University-Branch Damietta



Cellulose and cross-linked dextran (Sephadex) are used as the
solid supports and charged groups such as diethylaminoethyl (DEAE) or
carboxymethyl (CM) are chemically linked to them to give anion and
cation and the exchangers respectively. The preparation and commercial
availability of these materials beginning in the 1960 provided the biochemist
with powerful tools for separation of proteins and nucleic acid Figure 2
presents partial structures of DEAE-cellulose and CM cellulose

Figure 6.2. Partial structures of diethylaminoethyl-cellulose and carboxymethyl-
cellulose.


81
Chromatography Course Dr Ehab Aboueladab-Lecturer of Biochemistry-Mansoura University-Branch Damietta


The DEAE and CM groups are shown attached to the C
6
-hydroxyl group of
glucose. The DEAE and CM groups are also found attached to the hydroxyl
groups of C
2
and C
3
. The total degree of substitution of the DEAE and CM
groups must be less than one group per five glucose residues to maintain a
water-insoluble product.
Table 6.1. Pretreatment steps for DEAE-cellulose and CM -cellulose ion exchangers
Cellulose First treatment Intermediate
pH
Second treatment
DEAE 0.5 M HCl 4 0.5 M NaOH
CM 0.5 M NaOH 8 0.5 M HCl



82
Chromatography Course Dr Ehab Aboueladab-Lecturer of Biochemistry-Mansoura University-Branch Damietta



The dry ion-exchange celluloses are pretreated with acid and base to swell
the exchangers so that they become fully accessible to the charged
macromolecules in solution. The weighed exchanger is suspended in 15
volumes (w/v) of the "first treatment," acid or alkali depending on the
exchanger (Table. 1), and is allowed to stand at least 30 minutes but not
more than 2 hours. The supernatant is decanted and the exchanger is
washed until the effluent is at the "intermediate pH" The exchanger is stirred
into 15 volumes of the "second treatment" and allowed to stand for an
additional 30 minutes. The second treatment is repeated and the exchanger


83
Chromatography Course Dr Ehab Aboueladab-Lecturer of Biochemistry-Mansoura University-Branch Damietta


is washed with distilled water until the effluent is close to neutral pH. The
treated exchanger is placed into the acid component of the buffer (the pH
should be less than 4.5) and degassed under vacuum 10 cm Hg
pressure) with stirring, until bubbling stops The exchanger is then titrated
with the basic component of the buffer to the desired pH, filtered, and
suspended in fresh buffer to complete the pretreatment. The exchanger is
ter) above the
settled exchanger are removed by decantation. Buffer is added to the
exchanger so that the final volume of the slurry is l50% of the settled wet
volume of the exchanger. The column is then packed with the slurry of the
exchanger, the sample is applied, and elution is performed as described for
adsorption chromatography.
Three general methods are used for eluting molecules from the
exchanger:
(a) Changing the pH of the buffer to a value at which binding is weakened
(i.e., the pH is lowered for an anion exchanger and raised for a cation
exchanger),
(b) Increasing the ionic strength by increasing the concentration of salt in
the elution solvent, thereby weakening the electrostatic interactions between
the adsorbed molecule and the exchanger, and


84
Chromatography Course Dr Ehab Aboueladab-Lecturer of Biochemistry-Mansoura University-Branch Damietta


(c) Performing affinity elution. In affinity elution the adsorbed molecule is
usually a macromolecule that is desorbed from the affinity ligand by adding
a molecule that is charged and of opposite signs to the net charge on
the macromolecule and has a specific affinity for the macromolecule. Thus,
the reduction of the net charge on the macromolecule weakens its
electrostatic interaction with the exchanger sufficiently to permit the elution of
the macromolecule from the affinity ligand.
The stages of anion exchange chromatography.

An example of the use of ions exchange resins
Is the purification of Cytochrome C:


85
Chromatography Course Dr Ehab Aboueladab-Lecturer of Biochemistry-Mansoura University-Branch Damietta


Cytochrome C has an isoelectric point (pI) of 10.05; that is at pH
10.05 the number of positive charges will equal the number of negative
charges. A column containing a cation exchanger buffered, at pH 8.5, is
prepared. This column has a full negative charge. Cytochrome C at pH
8.5 has a full positive charge. An Impure solution of Cytochrome C at
pH 8.5 placed on the column, and water is passed through the
column (the pI of proteins is usually 7.0 or less) but Cytochrome C is
held firmly by electrostatic attraction to the resin heads. If the eluting
solvent pH is raised to about 10, the Cytochrome C will now has a
net zero charge and will pass rapidly through as a pure component
Summary Ion-exchange chromatography
Principle
Ion exchange chromatography separates molecules (proteins) according to
their differences between the overall charges. The proteins to be separated
must have a charge opposite to that of stationary phase in order to bind. Ion
exchange has two types according to the stationary phase charge:
1. Cation-exchanger: in which the stationary phase is charged
negatively in order to binds with positive molecules (cations)
2. Anion-exchanger: in which the stationary phase is charged positively
in order to binds with negative molecules (anions)


86
Chromatography Course Dr Ehab Aboueladab-Lecturer of Biochemistry-Mansoura University-Branch Damietta


A-Cation-exchange chromatography
Cation-exchange chromatography can be classified as: either strong or
weak. A strong cation exchanger contains strong acid which stable along
pH1-14. Whereas, weak cation exchanger contains weak acid which loss its
charge as the pH decrease below 4-5
The sample must be charged positive in order to bind with the negative
matrix (strong or weak acid). H+
B-Anion-exchange chromatography
Anion-exchange chromatography can be classified as: either strong or
weak. A strong anion exchanger contains strong base which stable along
pH1-14. Whereas, weak anion exchanger contains weak base which loss its
charge as the pH increase over 9
The sample must be charged negative in order to bind with the positive
matrix (strong or weak base).OH-
Components
1. The column containing the stationary phase (anion or cation
exchanger) on suitable matrix
2. Washing and eluting buffer
3. pump to withdrew the buffer
4. Detector


87
Chromatography Course Dr Ehab Aboueladab-Lecturer of Biochemistry-Mansoura University-Branch Damietta


Procedures
Before carry out the process, you must answer two important questions:
a) What is the sample charge? If +Ve, use cation exchanger. if Ve, use
anion exchanger
b) What is the suspected strength of the charge? If weak +Ve, use weak
cation exchanger, if strong +Ve, use strong cation exchanger, if weak Ve,
use weak anion exchanger, if strong Ve use strong anion exchanger.
e.g. the sample is weak negative proteins. So we will use anion exchanger
contain weak base.
1. (Loading step): the column is packed with the matrix that charged with
weak positive charge by adding weak base e.g. DEAE- cellulose (stationary
phase)
2. (Sampling step): apply the sample in the column: the negatively
charged proteins bind to positively charged matrix whereas; the positively
charged proteins flow down to the exterior. Some negative charged
contaminants can bind to matrix.
3. (Washing step): apply washing buffer (Tris-HCL) to remove the
contaminants remaining the target proteins.
4. (Elution step): now, we need to separate the target proteins from the
matrix, so we apply an eluting buffer that has the same charge of protein in


88
Chromatography Course Dr Ehab Aboueladab-Lecturer of Biochemistry-Mansoura University-Branch Damietta


order to substitutes it (ion exchange). Separation can be done also by ion
exclusion and ion pairing.
5. (Gradient step): make gradient elution with different buffer till you
obtain 100% correct proteins. i.e. repeat washing and eluting steps with
different buffer
6. (Detection step): after separation carry out detection by electrophoresis
Applications
1. Separation and detection of ions and ionized species.
2. Separation and purification of components from mixture
3. Identification of ionic impurities
Disadvantages
1. Analytes can be misidentified
2. Analytes are performed sequentially
3. Analysis consume eluent
Advantages
1. Selective to charge
2. Separation and detection of ions and ionized species
Relation between pH, Pi & ion exchange
pH: is quantitative description of the acidity of an aqueous solution.
pH= -Log [H]
+
=Log 1/[H]
+



89
Chromatography Course Dr Ehab Aboueladab-Lecturer of Biochemistry-Mansoura University-Branch Damietta


Isoelectric point (Pi): is the point at which the protein charge reach zero.

If pH >
P
I use anion exchange
If pH <
P
I use cation exchange
pH = pK + Log conjugated base / conjugated acid
Protein

------------Negative-------------------------o-----------------------Positive----------------------
(
p
H>pI) (
P
H=pI) (
p
H<pI)
Anion exchange resin Cation exchange resin
(Resin is positive A
+
) (Resin is negative C
-
)

Isoelectric point (pI)
At pH > pI, protein net charge is negative
At pH < pI, protein net charge is positive
At pH = pI, protein net charge is zero



90
Chromatography Course Dr Ehab Aboueladab-Lecturer of Biochemistry-Mansoura University-Branch Damietta


At pH > pI, use an anion exchange resin (positive resin)At pH <
pI, use a cation exchange resin (negative resin)




91
Chromatography Course Dr Ehab Aboueladab-Lecturer of Biochemistry-Mansoura University-Branch Damietta












92
Chromatography Course Dr Ehab Aboueladab-Lecturer of Biochemistry-Mansoura University-Branch Damietta


Chapter Seven
Desalting
Before an ion-exchange chromatographic step or after an ammonium
sulfate fractionation, it is usually necessary to remove the salt from the
solution of protein. Desalting is accomplished in one of two ways: dialysis
or gel filtration.

Dialysis is performed by filling a section of dialysis tubing (a semi
permeable membrane) having a sufficiently small molecular weight "Cut-off",
with the protein solution, and placing the filled tubing in a large volume of


93
Chromatography Course Dr Ehab Aboueladab-Lecturer of Biochemistry-Mansoura University-Branch Damietta


buffer. The decrease in salt concentration can be calculated easily from the
ratio of the volumes inside and outside of the bag.
Dialysis requires a few hours, after which the bag may be transferred
to fresh buffer if the reduction in salt concentration effected by one cycle is
deemed to be insufficient. In dialysis, all small molecules, including salt ions,
metal ions and cofactors, pass through the membrane, which retains only
macromolecules. Neither tightly bound metal ions and cofactors, nor
counterions to the macromolecule are effectively removed.
Since the initial solution in the bag is of much greater osmotic strength
than the surrounding buffer, the bag generally increases in volume. The
volume of the contents of the bag must be measured after dialysis if either
total protein or total enzyme units are to be calculated.
1. Ammonium Sulfate Fraction of Protein Mixtures
Increasing the salt concentration to a very high level will cause proteins
to precipitate from solution without denaturation if done in a gentle manner.
First, we want to understand why the protein precipitates. A protein in a
buffer solution is very highly hydrated, in other words, the ionic groups on the
surface of the protein attract and bind many water molecules very tightly:


94
Chromatography Course Dr Ehab Aboueladab-Lecturer of Biochemistry-Mansoura University-Branch Damietta




This graphic illustrates how proteins in solution are hydrated by water
molecules. When a lot of salt, such as ammonium sulfate, is added to the
protein solution, the salt ions attract the water molecules away from the
protein. This is partly since the salt ions have a much greater charge density
than the proteins. So as the salt is added and these small ions bind water
molecules, the protein molecules are forced to interact with themselves and
begin to aggregate:


95
Chromatography Course Dr Ehab Aboueladab-Lecturer of Biochemistry-Mansoura University-Branch Damietta




So when enough salt has been added, the proteins will be begin to
precipitate. If this is carried out at a cold temperature like in ice, the proteins
will precipitate without denaturation. Thus, the proteins can be collected by
centrifugation and then redissolved in solution using a buffer with low salt
content.
This process is called "Salting Out" and works best with divalent anions
like sulfate, especially ammonium sulfate which is highly soluble at ice
temperatures.
Salting out or ammonium sulfate precipitation is useful for concentrating
dilute solutions of proteins. It is also useful for fractionating a mixture of
proteins. Since large proteins tend to precipitate first, smaller ones will stay


96
Chromatography Course Dr Ehab Aboueladab-Lecturer of Biochemistry-Mansoura University-Branch Damietta


in solution. Thus, by analyzing various salt fractions, one can find conditions
where the protein you are studying precipitates and many of the other
proteins in the mixture stay in solution. The end result is that you are also
able to increase the purity of your protein of interest while you concentrate it
using an ammonium sulfate fractionation procedure.
2. Dialysis of Proteins
After a protein has been ammonium sulfate precipitate and taken back
up in buffer at a much greater protein concentration than before precipitation,
the solution will contain a lot of residual ammonium sulfate which was bound
to the protein. One way to remove this excess salt is to dialyze the protein
against a buffer low in salt concentration.





97
Chromatography Course Dr Ehab Aboueladab-Lecturer of Biochemistry-Mansoura University-Branch Damietta


This graphic illustrates the dialysis process. First, the concentrated
protein solution is placed in dialysis bag with small holes which allow water
and salt to pass out of the bag while protein is retained. Next the dialysis bag
is placed in a large volume of buffer and stirred for many hours (16 to 24
hours), which allow the solution inside the bag to equilibrate with the solution
outside the bag with respect to salt concentration. When this process of
equilibration is repeated several times (replacing the external solution with
low salt solution each time), the protein solution in the bag will reach a low
salt concentration:

The graphic illustrates the complete dialysis process, except for it
suggests you do this with distilled water. Really you want to do this process
with buffer to prevent the protein from denaturing due to the fact that distilled


98
Chromatography Course Dr Ehab Aboueladab-Lecturer of Biochemistry-Mansoura University-Branch Damietta


or deionized water is too low in salt and may have an undesirable pH for
your protein, which may cause it to denature.
In fact, dialysis is a good way to exchange the buffer the protein is in at the
same time you get rid of excess salt. For example, the GOT after ammonium
sulfate precipitation contains a mixture of buffers as well as excess salt. So
we use the buffer we want for the next step in the purification, which is ion-
exchange chromatography, as the external solution during dialysis. After the
dialysis process, the protein solution is dialyzed against the starting buffer for
the ion-exchange chromatography step, not only will the salt be removed but
the protein will now be in the buffer needed for the next step and ready to go.
Sometimes, proteins will precipitate during the dialysis process and
you will need to centrifuge the solution after dialysis to remove any particles
which would interfere with the next step such as ion-exchange
chromatography where particles would clog the column and prevent the
chromatography step from working. In addition, you may lose enzyme
activity during dialysis. So it is a good idea to keep some of your protein
solution as a sample before it is put in the dialysis bag so that it can be
assayed for enzyme activity before and after dialysis.
3. Alternative Methods for Desalting and Concentration of Proteins
There are several ways to get rid of excess salt in a protein solution.


99
Chromatography Course Dr Ehab Aboueladab-Lecturer of Biochemistry-Mansoura University-Branch Damietta


One rapid method is to use a small gel filtration column which contains a
gel with very small pores which will only allow water and salt inside the gel
particles and will exclude the protein. This method works very well and can
be done at 4C so that little or no enzyme activity is lost during processing. A
small amount of dilution of the protein solution will take place during
processing, but it is possible by this method to exchange the buffer and
prepare the protein solution.
Another way to both concentrate a protein and exchange the buffer, which
completely avoids precipitation, is called ultrafiltration:


Ultrafiltration is done a device which can withstand high pressure. First, the
ultrafiltration device is fitted with an ultrafilter membrane of the desired
molecular weight cut off such that you protein of interest will be retain in the


100
Chromatography Course Dr Ehab Aboueladab-Lecturer of Biochemistry-Mansoura University-Branch Damietta


cell. Next, the pressure cell is filled with the protein solution and nitrogen gas
at about 50 psi is applied while the cell is stirred gently at 4C. After about 1
hour, the solution will be decreased in volume usually without loss of activity.
To exchange the buffer the cell is filled with the desired buffer and the
concentration process are repeated.
Ion Exchange Chromatography
Since proteins have different net charge and charge distribution, ion
exchange chromatography can be an effective purification tool. For bench-
top preparations, usually gravity-flow columns are employed, but HPLC and
automated HPLC-like systems have grown in popularity. For gravity flow or
for use with low pressure peristaltic pumps, ion exchange media are usually
carbohydrate based. Charged groups are attached to solid supports (inert
phase) such as Sepharose, Sephadex and cellulose. Since these
carbohydrates are compressible, they are not used in higher-pressure
systems, and more rigid inert phases such as TSK (a polyether-coated gel)
are used. For higher pressures, reinforced Polysaccharides, and organically
coated silica (e.g., TSK) are used. The resins, especially poly
(styrenediviny1benzene) described by HIRS for use with enzymes were
used by MOORE and STEIN in their famous amino acid analyzer. They are
commonly employed for ion exchange chromatography of small molecules,


101
Chromatography Course Dr Ehab Aboueladab-Lecturer of Biochemistry-Mansoura University-Branch Damietta


but have given way to the ion exchange polysaccharides for preparative
applications in enzymology. The charged groups used with the solid
supports depend to some extent on the chemistry of the support material
itself, but are remarkably similar. Groups containing charged nitrogen atoms
are almost universally used for anion exchange media. These include, from
strong to weak, quaternary amino methyl or ethyl (QAE), tertiary amino
(diethylaminoethyl, DEAE, or diethylaminomethyl) and secondary plus
tertiary nitrogens (polyethylenimine, PEI). The quaternary amino compounds
are positively charged at any pH, but the others must be used at a pH below
the pK, of the protonated form (- 10, for DEAE). The conjugate base of the
strongly acidic sulfonic acid (i.e., alkyl or aryl sulfonate) and the weakly
acidic carboxylic acids (e.g., carboxymethyl, CM) are the most common
charged groups employed in cation exchangers. The carboxymethyl
packing must be used at a pH above their pK4. Methods for determining the
optimal pH for separation of proteins depends, of course, on the proteins.
Since most proteins are acidic, they are negatively charged at pH 7-8.
They therefore adsorb to a positively charged stationary phase to which
they act as counterions, providing that other anions are not available to play
the role of counterion. The cationic stationary phase is known as an anion
exchanger because it functions by exchanging one anionic counterion for


102
Chromatography Course Dr Ehab Aboueladab-Lecturer of Biochemistry-Mansoura University-Branch Damietta


another. Anionic proteins may bind more tightly to anion- exchange
stationary phases than simple salts because they possess more negative
charges than a simple anion. However, it is not the total charge on a protein,
but the charge density that determines the affinity. More precisely, it is the
charge distribution. Since a protein may interact with a stationary phase on
one side at a time, proteins with densely charged patches may be bound
more tightly. At pH values below the isoelectric point of a protein, the
net charge is positive, so negatively charged stationary phases (cation
exchange phases) are used. If a protein has an isoelectric point near
neutrality, either a cation exchange or an anion exchange system can
be used, depending on the pH employed. The important considerations in
choosing an optimal pH for separation of enzymes by ion exchange
chromatography have been reviewed. Protein solutions are generally
desalted, then loaded onto a column packed with a stationary phase having
the appropriate charge. Loading can often be done as rapidly as the columns
will flow without undue pressure; proteins that adsorb are retained at the top
of the column. As long as the capacity of the column is not exceeded, liters
of a (desalted, buffered) crude extract can be loaded onto a column of
modest size, so that a pre-chromatography concentration step is not needed.
After loading, the column is washed with the loading buffer to remove


103
Chromatography Course Dr Ehab Aboueladab-Lecturer of Biochemistry-Mansoura University-Branch Damietta


unabsorbed and weakly adsorbed proteins. The adsorbed proteins are then
eluted by washing the column with buffers of increasing salt
concentration (e.g., NaCl), which corresponds to increasing solvent
strength. This method of elution using a series of isocratic (constant
strength) elutions of progressively increasing strength is known as batch
elution. The ion having a charge of the same sign as the protein can act as
a displacing ion by competing for charged sites on the stationary phase. At
some concentration, the eluting ion competes effectively with the protein,
which accordingly, spends a larger fraction of its time in the mobile phase,
leading to elution. This concentration would be ideal to purify the protein of
interest providing that more loosely bound proteins were removed first,
because it affords the maximum discrimination among the charge
densities of the proteins still on the column. However, the protein might
elute as a broad, dilute band. A simple and common solution to elution is to
employ a linear concentration gradient of salt, such a gradient can cover a
range from 0 to 1 M NaCl over the volume of a few hundred ml to a few
liters, depending on the dimensions of the column and the steepness of the
gradient desired.


104
Chromatography Course Dr Ehab Aboueladab-Lecturer of Biochemistry-Mansoura University-Branch Damietta


A major advantage of gradient elution is that proteins having a wide
range of affinities for the column can be eluted in a single run. The
information obtained from a gradient elution may be used to determine an
Optimum salt concentration to be used in isocratic elution, but the procedure
is not straightforward. The theory of gradient elution is messy, even in the
simplest case. One egregious misstatement appears in numerous papers on
enzyme purification the enzyme elutes at such and such a concentration of
sodium chloride. Because the gradient travels much more rapidly in the
column than the protein (the protein is retained to some extent), the
concentration of sodium chloride in which the enzyme actually appears at
the bottom of the column is much higher than the concentration at which it
began to elute appreciably. Thus, the concentration in which it appears to
elute (concentration of sodium chloride in the fraction in which the activity
appears) is much too strong for use as an isocratic eluent. In addition, the
concentration in which the enzyme appears varies with the dimensions of the
column; longer columns cause the enzyme to appear to elute in a higher salt
concentration, simply because the gradient progresses as the enzyme
moves down the column. To exercise maximum control over the system, it is
useful to separate the effects of pH from those of ionic strength during ion
exchange chromatography. One of the ions involved in the buffering system


105
Chromatography Course Dr Ehab Aboueladab-Lecturer of Biochemistry-Mansoura University-Branch Damietta


bears the same charge as the protein and can therefore act as a displacing
ion. The concentration of this ion should not change with pH, so it should not
be the one involved in the equilibrium with solvent protons. Buffering ions
selected for use in ion exchange chromatography should have the same
charge as the column, i.e., cations for an anion exchange column, anions
for cation exchange. Hence, phosphate buffers are used for cation
exchange chromatography, and Tris (for instance) buffers are used for
anion exchange. It is necessary for the column to be completely
equilibrated with the starting solvent. Equilibration can be checked by
measurement of both pH and ionic strength (e.g., by conductivity) prior
to loading the column. Elution from an ion-exchange column could also be
accomplished using a change in pH. Stepwise pH changes are sometimes
employed, but do not generally produce high resolution of complex mixtures.
Reproducible continuous pH gradients are difficult to obtain because so
many of the components in the system engage in acid-base equilibria. A
workable system along these lines has been devised using a proprietary
mixed-bed packing and a multi-component buffer system to elute proteins at
their isoelectric pH. The process is called chromatofocusing because of a
loose analogy to isoelectric focusing gel electrophoresis.



106
Chromatography Course Dr Ehab Aboueladab-Lecturer of Biochemistry-Mansoura University-Branch Damietta


Chapter Eight
AFFINITY CHROMATOGRAPHY
Affinity chromatography is a specialized type of adsorption
chromatography in which a specific type of molecule is covalently linked to
an inert solid support. This specific molecule called a ligand, has a high
binding affinity for one of the compounds in a mixture of substances. The
process uses the unique biological property of the substance to bind to the
ligand specifically and reversibly and provides a high degree of selectivity in
the isolation and purification of biological molecules

Fig. 8.1. The steps of affinity chromatography


107
Chromatography Course Dr Ehab Aboueladab-Lecturer of Biochemistry-Mansoura University-Branch Damietta


A solution containing the substance to be purified. Usually a
macromolecule such as a protein (enzyme, antibody, hormone. etc.).
Polysaccharide or nucleic acid is passed through a column containing an
insoluble inert polymer to which the ligand has been covalently attached.
The ligand may be specific competitive inhibitors, substrate analogues,
product analogues, coenzymes and so on. Molecules in the mixture not
having affinity for the ligand pass through the column. Wide molecules that
have specific affinity for the ligand are bound and retained on the column.
The specifically adsorbed molecules) can be eluted by changing the ionic
strength the pH or by the addition of a competing ligand. In one
chromatographic step. The method is capable of isolating a single substance
in a pure form. It has thus become a powerful tool in the isolation and
purification of enzymes, antibodies, antigens, nucleic acids.
Polysaccharides, coenzyme or vitamin binding proteins, repressor proteins,
transport proteins, drug or hormone receptor structures and other
biochemical materials.
The Inert Support and the Ligand
The inert solid supports are the same materials discussed in the
preceding sections: cross-linked dextran cross-linked polyacrylamide,


108
Chromatography Course Dr Ehab Aboueladab-Lecturer of Biochemistry-Mansoura University-Branch Damietta


agarose and cellulose. The macromolecules to be separated should not be
retarded by a gel filtration process but should be retarded only by the
specific interaction with the ligand. The ligand must be a molecule that
display, special and unique affinity for the macromolecule to be purified it
also must have a chemical group that can be modified for covalent linkage to
the solid support without destroying or seriously decreasing its interaction
with the macromolecule to be purified. Also for successful affinity
chromatography, the chemical groups of the ligand that arc critical for the
binding of the macromolecule to be purified must be sufficiently distant from
the solid support to minimize steric interference with the binding process.
This steric problem has been solved by adding a long, hydrocarbon chain
spacer arm to the solid support and coupling the ligand to the end of the
arm. Alternatively the hydrocarbon arm may be attached to the ligand and
the arm attached to the solid support.
Attachment of the Ligand to the Solid Support
The polysaccharide solid supports-cross-linked dextran, agarose, and
cellulose can be activated by reaction with alkaline cyanogen bromide. The
products that arc formed upon coupling of the activated polysaccharides with


109
Chromatography Course Dr Ehab Aboueladab-Lecturer of Biochemistry-Mansoura University-Branch Damietta


amino compounds are derivatives of amino carbonic acid. The reactions are
the following:

If the ligand contains an amino group, it can be coupled directly to the
activated polysaccharide. A spacer arm can be introduced by sequential
reaction with a diaminoalkane and glutaraldehyde. The amino group on the
ligand can then be coupled to the free aldehyde group.


110
Chromatography Course Dr Ehab Aboueladab-Lecturer of Biochemistry-Mansoura University-Branch Damietta



If the ligand contains an aldehyde group instead of an amino group, it
can be coupled directly to the free amino group of the diaminoalkane.
Ligands may be coupled to polyacrylamide by displacing the amide group of
the polyacrylamide by heating with a diaminoalkane (c), followed by reaction
with glutaraldehyde (d).




111
Chromatography Course Dr Ehab Aboueladab-Lecturer of Biochemistry-Mansoura University-Branch Damietta


The Schiff base that results from the reaction of glutaraldehyde with an
amino group may be stabilized by reduction with sodium cyanoborohydride
without affecting the aldehyde group. The ligand can then be coupled to the
aldehyde group.

Another method of activating polyacrylamide is to form the hydrazide
derivative by reaction with hydrazine hydrate. When an amino, aldehyde, or
hydrazide group is incorporated onto the solid support, the support becomes
activated so that ligands may be attached through amino, carboxyl, phenolic,
or imidazole groups.








112
Chromatography Course Dr Ehab Aboueladab-Lecturer of Biochemistry-Mansoura University-Branch Damietta


Chapter Nine
Gel electrophoresis
The movement of a charged presented by Equation 1.0 subjected to an
electric field:
(Equation 1.0)
where
E = the electric field in volts/cm
q = the net charge on the molecule
f = frictional coefficient, which depends on the mass and shape of the
molecule
V = the velocity of the molecule
The charged particle moves at a velocity that depends directly on the
electrical field (E) and charge (q) but inversely on a counteracting force
generated by the viscous drag (f ) The applied voltage represented by E in
Equation 1.0 is usually held constant during electrophoresis, although some
experiments are run under conditions of constant current (where the voltage


113
Chromatography Course Dr Ehab Aboueladab-Lecturer of Biochemistry-Mansoura University-Branch Damietta


changes with resistance) or constant power (the product of voltage and
current). Under constant-voltage conditions, Equation 1.0 shows that the
movement of a charged molecule depends only on the ratio q/f. For
molecules of similar conformation (for example, a collection of linear DNA
fragments or spherical proteins), varies with size but not shape; therefore,
the only remaining variables in Equation 1.0 are the charge (q) and mass
dependence of (f ) meaning that under such conditions molecules migrate in
an electric field at a rate proportional to their charge-to-mass ratio. The
movement of a charged particle in an electric field is often defined in terms of
mobility, , the velocity per unit of electric field (Equation 2.0).
(Equation 2.0)
This equation can be modified using Equation 1.0.
(Equation 3.0)
In theory, if the net charge, (q), on a molecule is known, it should be possible
to measure (f) and obtain information about the hydrodynamic size and
shape of that molecule by investigating its mobility in an electric field.


114
Chromatography Course Dr Ehab Aboueladab-Lecturer of Biochemistry-Mansoura University-Branch Damietta


Attempts to define (f) by electrophoresis have not been successful,
primarily because Equation 3.0 does not adequately describe the
electrophoretic process. Important factors that are not accounted for in the
equation are interaction of migrating molecules with the support medium and
shielding of the molecules by buffer ions. This means that electrophoresis is
not useful for describing specific details about the shape of a molecule.
Instead, it has been applied to the analysis of purity and size of
macromolecules. Each molecule in a mixture is expected to have a unique
charge and size, and its mobility in an electric field will therefore be unique.
This expectation forms the basis for analysis and separation by all
electrophoretic methods The technique is especially useful for the analysis of
ammo acids, peptides, proteins, nucleotides, nucleic acids, and other
charged molecules.
Method of Electrophoresis
All types of electrophoresis are based on the principles just outlined.
The major difference between methods is the type of support medium, which
can be either cellulose or thin gels. Cellulose is used as a support medium
for low-molecular-weight biochemical such as ammo acids and
carbohydrates, and polyacrylamide and agarose gels are widely used as


115
Chromatography Course Dr Ehab Aboueladab-Lecturer of Biochemistry-Mansoura University-Branch Damietta


support media for larger molecules. Geometries (vertical and horizontal),
buffers, and electrophoretic conditions for these two types of gels provide
several different experimental arrangements, as described below.
Polyacrylamide Gel Electrophoresis (PAGE)
Gels formed by polymerization of acrylamide have several positive
features in electrophoresis:
A) High resolving power for small and moderately sized proteins and
nucleic acids (up to approximately 1 X 10
6
daltons),
B) Acceptance of relatively large sample sizes,
C) Minimal interactions of the migrating molecules with the matrix, and
D) Physical stability of the matrix that gels can be prepared with
different pore sizes by changing the concentration of cross-linking
agents. Electrophoresis through polyacrylamide gels leads to enhanced
resolution of sample components because the separation is based on both
molecular sieving and electrophoretic mobility The order of molecular
movement in gel filtration and PAGE is very different, however in gel
filtration, large molecules migrate through the matrix faster than small
molecules The opposite is the case for gel electrophoresis, where there is


116
Chromatography Course Dr Ehab Aboueladab-Lecturer of Biochemistry-Mansoura University-Branch Damietta


no void volume in the matrix, only a continuous network of pores
throughout the gel. The electrophoresis gel is comparable to a single bead in
gel filtration. Therefore, large molecules do not move easily through the
medium, and the rate of movement is small molecules followed by
large molecules.
Polyacrylamide gels are prepared by the free radical polymerization of
acrylamide and the cross-linking agent N,N'- methylene-bis-acrylamide.
Chemical polymerization is controlled by an initiator-catalyst system,
ammonium persulfate-N,N,N
\
,N
\
tetramethylethylenediamine (TEMED).
Photochemical polymerization may be initiated by riboflavin in the presence
of ultraviolet (UV) radiation. A standard gel for protein separation is 7.5%
polyacrylamide. It can be used over the molecular size range of 10,000 to
1,000,000 daltons; however, the best resolution is obtained in the range of
30,000 to 300,000 daltons. The resolving power and molecular size range of
a gel depend on the concentrations of acrylamide and bis-acrylamide Lower
concentrations give gels with larger pores, allowing analysis of higher-
molecular-weight biomolecules In contrast, higher concentrations of
acrylamide give gels with smaller pores, allowing analysis of lower-
molecular-weight biomolecules


117
Chromatography Course Dr Ehab Aboueladab-Lecturer of Biochemistry-Mansoura University-Branch Damietta



(Table 9.1) Effective Range of Separation of DNA by PAGE
Acrylamide
(% W/V)
Range of Separation
(bp)
Bromophenol
Blue
Xylene Cyanol
35 1000-2000 100 450
50 80-500 65 250
80 60-400 50 150
120 40-200 20 75
200 5-100 10 50


118
Chromatography Course Dr Ehab Aboueladab-Lecturer of Biochemistry-Mansoura University-Branch Damietta


Polyacrylamide electrophoresis can be done using either of two
arrangements, column or slab. Figure 9.1 shows the typical arrangement for
a column gel. Glass tubes (10 cm X 6 mm l.d.) are filled with a mixture of
acrylamide, N,N'-methylene-bis-acrylamide, buffer, and free radical initiator
catalyst. Polymerization occurs in 30 to 40 minutes. The gel column is
inserted between two separate buffer reservoirs. The upper reservoir usually
contains the cathode and the lower the anode. Gel electrophoresis is usually
carried out at basic pH, where most biological polymers are anionic; hence,
they move down toward the anode. The sample to be analyzed is layered on
top of the gel and voltage is applied to the system. A "tracking dye" is also
applied, which moves more rapidly through the gel than the sample
components. When the dye band has moved to the opposite end of the
column, the voltage is turned off and the gel is removed from the column and
stained with a dye. Chambers or column gel electrophoresis is commercially
available or can be constructed from inexpensive materials.


119
Chromatography Course Dr Ehab Aboueladab-Lecturer of Biochemistry-Mansoura University-Branch Damietta



Slab gels are now more widely used than column gels. A slab gel on
which several samples may be analyzed is more convenient to make and
use than several individual column gels. Slab gels also offer the advantage
that all samples are analyzed m a matrix environment that is identical in
composition. A typical vertical slab gel apparatus is shown in Figure 9.2.


120
Chromatography Course Dr Ehab Aboueladab-Lecturer of Biochemistry-Mansoura University-Branch Damietta



The polyacrylamide slab is prepared between two glass plates that are
separated by spacers Figure 9.3.

The spacers allow a uniform slab thickness of 0.5 to 2.0 mm, which is
appropriate for analytical procedures. Slab gels are usually 8 X 10 cm or 10
X 10 cm, but for nucleotide sequencing, slab gels as large as 20 X 40 cm are


121
Chromatography Course Dr Ehab Aboueladab-Lecturer of Biochemistry-Mansoura University-Branch Damietta


often required. A plastic "comb" inserted into the top of the slab gel during
polymerization forms indentations in the gel that serve as sample wells. Up
to 20 sample wells may be formed. After polymerization, the comb is
carefully removed and the wells are rinsed thoroughly with buffer to remove
salts and any unpolymerized acrylamide. The gel plate is clamped into place
between two buffer reservoirs, a sample is loaded into each well, and
voltage is applied. For visualization, the slab is removed and stained with an
appropriate dye.
Perhaps the most difficult and inconvenient aspect of polyacrylamide
gel electrophoresis is the preparation of gels. The monomer, acrylamide, is a
neurotoxin and a cancer suspect agent; hence, special handling is required.
Other necessary reagents including catalysts and initiators also require
special handling and are unstable- In addition, it is difficult to make gels that
have reproducible thicknesses and compositions. Many researchers are now
turning to the use of precast polyacrylamide gels. Several manufacturers
now offer gels precast in glass or plastic cassettes. Gels for all experimental
operations are available including single percentage (between 3 and 27%) or
gradient gel concentrations and a variety or sample well configurations and


122
Chromatography Course Dr Ehab Aboueladab-Lecturer of Biochemistry-Mansoura University-Branch Damietta


buffer chemistries. Several modifications of PAGE have greatly increased its
versatility and usefulness as an analytical tool.
Sodium Dodecyl Sulfate-Polyacrylamide Gel Electrophorosis
(SDS-PAGE)
The electrophoretic techniques previously discussed are not applicable
to the measurement of the molecular weights of biological molecules
because mobility is influenced by both charge and size. If protein samples
are treated so that they have a uniform charge, electrophoretic mobility then
depends primarily on size (see Equation 2.0). The molecular weights of
proteins may be estimated if they are subjected to electrophoresis in the
presence of a detergent, sodium dodecyl sulfate (SDS), and a disulfide bond
reducing agent, mercaptoethanol. This method is often called "denaturing
electrophoresis." When protein molecules are treated with SDS, the
detergent disrupts the secondary, tertiary, and quaternary structure to
produce linear polypeptide chains coated with negatively charged SDS
molecules. The presence of mercaptoethanol assists in protein denaturation
by reducing all disulfide bonds. The detergent binds to hydrophobic regions
of the denatured protein chain in a constant ratio of about 14 g of SDS per


123
Chromatography Course Dr Ehab Aboueladab-Lecturer of Biochemistry-Mansoura University-Branch Damietta


gram of protein. The bound detergent molecules carrying negative charges
mask the native charge of the protein In essence, polypeptide chains of a
constant charge/mass ratio and uniform shape are produced The
electrophoretic mobility of the SDS-protein complexes is influenced primarily
by molecular size the larger molecules are retarded by the molecular sieving
effect of the gel, and the smaller molecules have greater mobility Empirical
measurements have shown a linear relationship between the log molecular
weight and the electrophoretic mobility Figure 9.4



124
Chromatography Course Dr Ehab Aboueladab-Lecturer of Biochemistry-Mansoura University-Branch Damietta


In practice, a protein of unknown molecular weight and subunit
structure is treated with 1% SDS and 0.1 M mercaptoethanol in
electrophoresis buffer. A standard mixture of proteins with known
molecular weights must also be subjected to electrophoresis under the same
conditions. Two sets on standards are commercially available, one for low-
molecular-weight proteins (molecular weight range 14,000 to 100,000) and
one for high-molecular weight proteins D5,000 to 200,000) Figure 9.5

a stained gel after electrophoresis of a standard protein mixture After
electrophoresis and dye staining, mobilities are measured and molecular


125
Chromatography Course Dr Ehab Aboueladab-Lecturer of Biochemistry-Mansoura University-Branch Damietta


weights determined graphically SDS-PAGE is valuable for estimating the
molecular weight of protein subunits This modification of gel electrophoresis
finds its greatest use in characterizing the sizes and different types of
subunits in oligomeric proteins. SDS-PAGE is limited to a molecular
weight range of 10,000 to 200,000. Gels of less than 2.5% acrylamide
must be used for determining molecular weights above 200,000, but
these gels do not set well and are very fragile because of minimal cross-
linking. A modification using gels of agarose-acrylamide mixtures allows the
measurement of molecular weights above 200,000.
Agarose Gel Electrophoresis
The electrophoretic techniques discussed up to this point are useful for
analyzing proteins and small fragments of nucleic acids up to 350,000
daltons (500 bp) in molecular size; however, the small pore sizes in the gel
are not appropriate for analysis of large nucleic acid fragments or intact DNA
molecules. The standard method used to characterize RNA and DNA in the
range 200 to 50,000 base pairs 50 kilobases) is electrophoresis with agarose
as the support medium.
Agarose, a product extracted from seaweed, is a linear polymer of
galactopyranose derivatives. Gels are prepared by dissolving agarose in


126
Chromatography Course Dr Ehab Aboueladab-Lecturer of Biochemistry-Mansoura University-Branch Damietta


warm electrophoresis buffer. After cooling the gel mixture to 50C, the
agarose solution is poured between glass plates as described for
polyacrylamide. Gels with less than 0.5% agarose are rather fragile and
must be used in a horizontal arrangement (Figure 4.8). The sample to be
separated is placed in a sample well made with a comb, and voltage is
applied until separation is complete. Precast agarose gels of all shapes,
sizes, and percent composition are commercially available. Nucleic acids
can be visualized on the slab gel after separation by soaking in a solution of
ethidium bromide, a dye that displays enhanced fluorescence when
intercalated between stacked nucleic acid bases. Ethidium bromide may be
added directly to the agarose solution before gel formation. This method
allows monitoring of nucleic acids during electrophoresis. Irradiation of
ethidium bromide treated gels by UV light results in
orange-red bands where nucleic acids are present. The mobility of nucleic
acids in agarose gels is influenced by the agarose concentration and the
molecular size and molecular conformation of the nucleic acid. Agarose
concentrations of 0.3 to 2.0% are most effective for nucleic acid separation
Table 9.2


127
Chromatography Course Dr Ehab Aboueladab-Lecturer of Biochemistry-Mansoura University-Branch Damietta



Figure 9.6

The separation of DNA fragments on agarose gels. Like proteins,
nucleic acids migrate at a rate that is inversely proportional to the
logarithm of their molecular weights; hence, molecular weights can be
estimated from electrophoresis results using standard nucleic acids or
DNA fragments of known molecular weight. The DNA conformations most
frequently encountered are superhelical circular (form I), nicked circular


128
Chromatography Course Dr Ehab Aboueladab-Lecturer of Biochemistry-Mansoura University-Branch Damietta


(form II), and linear (form III). The small, compact, supercoiled form I
molecules usually have the greatest mobility, followed by the rodlike, linear
form III molecules. The extended, circular form II molecules migrate more
slowly. The relative electrophoretic mobility of the three forms of DNA,
however, depends on experimental conditions such as agarose
concentration and ionic strength.
Isoelectric Focusing of Proteins
Another important and effective use of electrophoresis for the analysis of
Proteins are isoelectric focusing (IEF), which examines electrophoretic
mobility as a function of pH. The net charge on a protein is pH dependent.
Proteins below their isoelectric pH (pH
I
or the pH at which they have
zero net charge) are positively charged and migrate to a medium of
fixed pH toward the negatively charged cathode at a pH above its
isoelectric point, a protein is deprotonated and negatively charged and
migrates toward the anode If the pH of the electrophoretic medium is
identical to the pH
I
of a protein, the protein has a net charge of zero and
does not migrate toward either electrode. Theoretically, it should be possible
to separate protein molecules and to estimate the pH: of a protein by
investigating the electrophoretic mobility in a series of separate experiments


129
Chromatography Course Dr Ehab Aboueladab-Lecturer of Biochemistry-Mansoura University-Branch Damietta


in which the pH of the medium is changed. The pH at which there is no
protein migration should coincide with the pH
I
of the protein. Because such a
repetitive series determine the pH
I
, IEF has evolved as an alternative
method for performing a single electrophoresis run in a medium of gradually
changing pH.
Figure 9.7

illustrates the construction and operation of an IEF pH gradient. An acid,
usually phosphoric, is placed at the cathode; a base, such as
triethanolamine, is placed at the anode. Between the electrodes is a medium
in which the pH gradually increases from 2 to 10. The pH gradient can be
formed before electrophoresis is conducted or formed during the course of
electrophoresis. The pH gradient can be either broad (pH 2-10) for


130
Chromatography Course Dr Ehab Aboueladab-Lecturer of Biochemistry-Mansoura University-Branch Damietta


separating several proteins of widely ranging pH
I
values or narrow (pH 7-8)
for precise determination of the pH
I
of a single protein. P in Figure 7.0
represents different molecules of the same protein in two different regions of
the pH gradient. Assuming that the pH in region 1 is less than the pH
I
of the
protein and the pH in region 2 is greater than the pH
I
of the protein,
molecules of P in region 1 will be positively charged and will migrate m an
applied electric field toward the cathode. As P migrates, it will encounter an
increasing pH, which will influence its net charge. As it migrates up the pH
gradient, P will become increasingly deprotonated and its net charge will
decrease toward zero. When P reaches a region where it's net charge is
zero (region 3), it will stop migrating.
Illustration of isoelectric a surface microelectrode, or the position of the
protein can be compared to that of a calibration set of proteins of bown pH
values. P molecules in region 2 will be negatively charged and will migrate
toward the anode. In this case, the net charge on P molecules will gradually
decrease to zero as P moves down the pH gradient, and P molecules
originally in region 2 will approach region 3 and come to rest. The P
molecules move in opposite directions, but the final outcome of IEF is that P
molecules located anywhere m the gradient will migrate toward the region


131
Chromatography Course Dr Ehab Aboueladab-Lecturer of Biochemistry-Mansoura University-Branch Damietta


corresponding to their isoelectric point and will eventually come to rest in a
sharp band; that is, they will "focus" at a point corresponding to their pH
I
.
Since different protein molecules in mixtures have different pH
I
values, it is
possible to use IEF to separate proteins In addition; the pH
I
of each protein
in the mixture can be determined by measuring the pH of the region where
the protein is focused. The pH gradient is prepared in a horizontal glass tube
or slab. Special precautions must be taken so that the pH gradient remains
stable and is not disrupted by diffusion or convective mixing during the
electrophoresis experiment. The most common stabilizing technique is to
form the gradient in a polyacrylamide, agarose, or dextran gel. The pH
gradient is formed in the gel by electrophoresis of synthetic polyelectrolyte,
called ampholytes, which migrate to the region of their pH
I
values just as
proteins do and establish a pH gradient that is stable for the duration of the
IEF run. Ampholytes are low-molecular-weight polymers that have a wide
range of isoelectric points because of their numerous ammo and carboxyl or
sulfonic acid groups. The polymer mixtures are available in specific pH
ranges (pH 5-7, 6-8, and 3.5-10, etc.). It is critical to select the appropriate
pH range for the ampholyte so that the proteins to be studied have pH
I

values in that range. The best resolution is, of course, achieved with an
ampholyte mixture over a small pH range (about two units) encompassing


132
Chromatography Course Dr Ehab Aboueladab-Lecturer of Biochemistry-Mansoura University-Branch Damietta


the pH
I
of the sample proteins. If the pH
I
values for the proteins under study
are unknown, an ampholyte of wide pH range (pH 3-10) should be used first
and then a narrower pH range selected for use. The gel medium is prepared
as previously described except that the appropriate ampholyte is mixed prior
to polymerization. The gel mixture is poured into the desired form (column
tubes, horizontal slabs, etc.) and allowed to set. Immediately after casting of
the gel, the pH is constant throughout the medium, but application of voltage
will induce migration of ampholyte molecules to form the pH gradient. The
standard gel for proteins with molecular sizes up to 100,000 daltons is 7.5%
polyacrylamide; however, if larger proteins are of interest, gels with larger
pore sizes must be prepared. Such gels can be prepared with a lower
concentration of acrylamide (about 2%) and 0.5 to 1% agarose to add
strength. Precast gels for isoelectric focusing are also commercially
available. The protein sample can be loaded on the gel in either of two ways.
A concentrated, salt-free sample can be layered on top of the gel as
previously described for ordinary gel electrophoresis. Alternatively, the
protein can be added directly to the gel preparation, resulting in an even
distribution of protein throughout the medium. The protein molecules move
more slowly than the low-molecular-weight ampholyte molecules, so the pH
gradient is established before significant migration of the proteins occurs.


133
Chromatography Course Dr Ehab Aboueladab-Lecturer of Biochemistry-Mansoura University-Branch Damietta


Very small protein samples can be separated by IER. For analytical
purposes, 10 to 50 g is a typical sample size. Larger sample sizes (up to 20
mg) can be used for preparative purposes.





134
Chromatography Course Dr Ehab Aboueladab-Lecturer of Biochemistry-Mansoura University-Branch Damietta








135
Chromatography Course Dr Ehab Aboueladab-Lecturer of Biochemistry-Mansoura University-Branch Damietta




The polymerization reaction of acrylamide and methylenebisacrylamide



136
Chromatography Course Dr Ehab Aboueladab-Lecturer of Biochemistry-Mansoura University-Branch Damietta






137
Chromatography Course Dr Ehab Aboueladab-Lecturer of Biochemistry-Mansoura University-Branch Damietta






138
Chromatography Course Dr Ehab Aboueladab-Lecturer of Biochemistry-Mansoura University-Branch Damietta


Chapter Ten
Question
Consider the following TLC plate when answering questions (1) and (2). The
plate was developed in hexanes.

1) The R
f
of Compound C is: a) 1.0 b) 0.8 c) 0.5 d) 3.5
2) A fresh TLC plate is spotted with compounds A, B, and C, but this time the
solvent is isopropanol. How will this affect the R
f
of compound A?
a) The R
f
of A in the new solvent will be the same as in hexanes
b) The R
f
of A in the new solvent will be greater than that in hexanes
c) The R
f
of A in the new solvent will be less than that in hexanes
3) The TLC plate is prepared by drawing a light line 1 cm from the bottom of
the plate to mark where the pain relievers should be spotted. This line is
drawn with: a) a pencil b) a black pen c) a blue pen d) a red pen
4)Which of the following methods can be used to visualize the spots on a
developed TLC plate?
a) spraying the plate with a reagent b) iodine (I
2
) crystals in a jar
c) visualization by the color of the compound d) all of the above


5) The three compounds below are run on a silica gel TLC plate developed
in hexanes-ethyl acetate (70:30).


139
Chromatography Course Dr Ehab Aboueladab-Lecturer of Biochemistry-Mansoura University-Branch Damietta



a) The carboxylic acid will run the fastest and the alcohol the slowest
b) The aromatic will run the fastest and the carboxylic acid the slowest
c) The alcohol will run the fastest and the aromatic the slowest
6) Which of the following solvent mixtures is more polar?
a) ethyl acetate/hexanes 50:50 b) ethyl acetate/hexanes 80:20
c) ethyl acetate/hexanes 20:80 d) all of the above
7) If two substances are run on the same TLC plate have the same R
f
value,
they are:
a) Likely, but not necessarily, the same compound
b) Definitely the same compound
c) Likely, but not necessarily, different compounds
d) Definitely different compounds
8-Amino acid analyzers are instruments that automatically separate amino
acids by cation exchange chromatography. Predict the order of elution (first
to last) for each of the following sets of amino acids at pH = 4.
(a) Gly, Asp, His
(b) Arg, Glu, Ala
(c) Phe, His, Glu

9-Predict the relative order of paper chromatography Rf values for the amino
acids in the following mixture: Ser, Lys, Leu, Val, and Ala. Assume that the
developing solvent is n-butanol, water, and acetic acid.




140
Chromatography Course Dr Ehab Aboueladab-Lecturer of Biochemistry-Mansoura University-Branch Damietta


10-In what order would the following proteins be eluted from a DEAE-
Cellulose ion exchanger by an increasing salt gradient. The pH1 is listed
for each protein.
Egg albumin, 4.6 , Pepsin, 1.0 , Serum albumin, 4.9
Cytochrome c, 10.6, Myoglobin, 6.8, Hemoglobin, 6.8

11-Draw the elution curve (Abs 280 nm vs. fraction number) obtained by
passing a mixture of the following proteins through a column of Sephadex G-
100. The molecular weight is given for each protein.
Myoglobin, 16,900 Myosin, 524,000
Catalase, 222,000 Serum albumin, 68,500
Cytochrome c, 13,370 Chymotrypsinogen, 23,240

1-Answer: (c) R
f
is the distance the spot travels divided by how far the
solvent travels. In this case, the answer is 5 cm/10 cm, or 0.5.
2-Answer: (b) isopropanol is more polar and a stronger eluter than is
hexanes, and thus will carry compound A and also compounds B and C
farther along the plate than will hexanes. (Remember: all compounds travel
faster when a polar solvent is used.)
3- Answer: (a) Always use a pencil to mark a TLC plate. If a pen is used, the
pigments in the ink will move up the plate as the plate is developed; pencil
lines are not so affected
4- Answer: (d) All of the above. In the TLC experiment, the compounds will
be visualized by their color
5-Answer: (c) The eluting order of these three classes of compounds
fastest (highest Rf) to slowest (lowest Rf) is: (less polar) aromatic >


141
Chromatography Course Dr Ehab Aboueladab-Lecturer of Biochemistry-Mansoura University-Branch Damietta


alcohol > carboxylic acid (most polar). This is because carboxylic
acid are the most polar and aromatics the least polar of the three
types of compounds. (Remember: all compounds travel faster when a
polar solvent is used.)
6- Answer: (b) Ethyl acetate (an ester) is more polar than hexanes (a
hydrocarbon solvent = non plolar). In (b), the solvent has a greater
percentage of this more polar solvent than in (a).
7- Answer: (a) Compounds which have the same Rf value when run on the
same TLC plate are likely the same compound. This is especially true when
the TLC plate is run of a reaction mixture: in such a case the chemist usually
has a pretty good idea of what compounds might be present, and thus
chooses and runs appropriate standards. In this situation, if an "unknown"
has the same Rf as a standard, it's a pretty good indication that the
compound is the same as the standard. However, if the entire organic
compounds is considered, many different compounds will have the same Rf
in a particular TLC system, and thus Rf values as a means of compound
identification is limited.
8. (a) Asp, Gly, His
(b) Glu, Ala, Arg
(c) Glu, Phe, His
9. Ser, Lys, Ala, Val, Leu
10. Cyt c, myoglobin = hemoglobin, serum albumin, egg albumin, pepsin
11. Myosin, catalase, serum albumin, chymotrypsinogen, myoglobin,
Cytochrome c




142
Chromatography Course Dr Ehab Aboueladab-Lecturer of Biochemistry-Mansoura University-Branch Damietta


12-Name the techniques described for separating cellular proteins:

a) Taking advantage of unique structural or functional properties of a protein,
this technique specifically removes the protein of interest from a solution.

b) Proteins leave the mobile phase, associating with a negatively charged
immobile structure, such as bead or resin.

c) Proteins are separated on the basis of their ability to migrate in an electric
field, an indicator of relative size.

d) Proteins are chromatographically separated on the basis of size.

Answer
a- Affinity chromatography
b- Cation exchange chromatography
c- Electrophoresis
d- Gel filtration/ size-exclusion chromatography
=========================================================
13-Amino acid analyzers are instruments that automatically separate amino
acids by cation-exchange chromatography. Predict the order of elution
(first to last) for each of the following sets of amino acids at pH = 4.
(a) Gly, Asp, His
(b) Arg, Glu, Ala
(c) Phe, His, Glu

14-Predict the relative order of paper chromatography R
f
values for the
Amino acids in the following mixture: Ser, Lys, Leu, Val, and Ala.
Assume that the developing solvent is n-butanol, water, and acetic acid.

15-In what order would the following proteins be eluted from a DEAE-
cellulose ion exchanger by an increasing salt gradient. The pH1 is listed
for each protein.
Egg albumin, 4.6 , Pepsin, 1.0 , Serum albumin, 4.9
Cytochrome c, 10.6, Myoglobin, 6.8, Hemoglobin, 6.8

16-Draw the elution curve (Abs 280 nm vs. fraction number) obtained by
passing a mixture of the following proteins through a column of Sephadex G-
100. The molecular weight is given for each protein.


143
Chromatography Course Dr Ehab Aboueladab-Lecturer of Biochemistry-Mansoura University-Branch Damietta


Myoglobin, 16,900 Myosin, 524,000
Catalase, 222,000 Serum albumin, 68,500
Cytochrome c, 13,370 Chymotrypsinogen, 23,240

Answer
13. (a) Asp, Gly, His
(b) Glu, Ala, Arg
(c) Glu, Phe, His

14. Ser, Lys, Ala, Val, Leu

15. Cyt c, myoglobin = hemoglobin, serum albumin, egg albumin, pepsin

16. Myosin, catalase, serum albumin, chymotrypsinogen, myoglobin,
Cytochrome c
======================================================
17-What physical characteristics of a biomolecule influence its rate of
movement in an electrophoresis matrix?

18. Draw a slab gel to show the results of nondenatunng electrophoresis of
the following mixture of proteins. The molecular weight is given for each.
Lysozyme A3,930) Egg white albumin D5,000)
Chymotrypsin B1,600) Serum albumin F5,400)

19. Each of the proteins listed below is treated with sodium dodecyl sulfate
and separated by electrophoresis on a polyacrylamide slab gel. Draw
pictures of the final results.
(a) Myoglobm
(b) Hemoglobin (two a subunits, molecular weight = 15,500; two /3
subumts, molecular weight = 16,000)

20. Explain the purpose of each of the chemical reagents that are used for
PAGE.
(a) acrylamide (d) sodium dodecyl sulfate
(b) N, /V'-methylene-bis-acrylamide (e) Coomassie Blue dye
(c) TEMED (f) bromophenol blue

21. What is the main advantage of slab gels over column gels for PAGE?
22. Is it possible to use polyacrylamide as a matrix for electrophoresis of


144
Chromatography Course Dr Ehab Aboueladab-Lecturer of Biochemistry-Mansoura University-Branch Damietta


Nucleic acids? What are the limitations, if any?
23. Explain the purposes of protein and nucleic acid "blotting."
24. Can polyacrylamide gels be used for the analysis of plasmid DNA with
greater than 3000 base pairs? Why or why not?
25. Describe the toxic characteristics of acrylamide and outline precautions
necessary for its use.

Answer
17. Charge, size
18. From top to bottom: serum albumin, egg white albumin, chymotrypsin,
lysozyme
19. (a) Monomer for polymeric gel matrix
(b) Monomer for adding cross-linking to gel matrix
(c) Catalyst for polymerization process.
(d) Detergent that denatures proteins for electrophoresis
(e) Dye used to stain proteins after gel electrophoresis
(f) Molecule used as a "tracking dye" during electrophoresis
20. The gel matrix in slab gels is more uniform than column gels, which are
made individually.
21. Polyacrylamide gels may be used for nucleic acids up to 2000 base
pairs.
========================================================
(1) Why is it important to avoid air bubbles in the column during
chromatography?
(a) the air in the bubbles might react with the compounds being
separated
(b) bubbles are toxic and harmful to your health
(c) bubbles cause the samples to travel unevenly down the column and
thus the components might not separate
(d) bubbles block the flow from the bottom of the column
(2) Spinach is green, however the pigments it contains are green,
orange, yellow,
and grey. Which of the following statements is true?
(a) Chlorophyll is grey and carotene is yellow-orange.
(b) Chlorophyll is green and carotene is yellow-orange.
(c) Chlorophyll is grey and carotene is green.
(3) A column chromatography procedure for the separation of a polar
and a


145
Chromatography Course Dr Ehab Aboueladab-Lecturer of Biochemistry-Mansoura University-Branch Damietta


non-polar compound calls for sequential elution with the following
two solvents:
methylene chloride
hexanes
Which solvent should be used first?
(a) methylene chloride
(b) hexanes
(c) it does not matter which solvent is used first
(4) What happens if you load the spinach pigment mixture onto the
column in
too much methylene chloride?
(a) the spinach pigments will not separate into individual components
because they will travel rapidly down the column
(b) the spinach pigments will evaporate
(c) too much methylene chloride in the loading solvent will not affect
the separation
(5) Of the following compounds, which will be the first to elute from an
alumina chromatography column? Last?

(a) a will elute first and b will elute last
(b) d will elute first and c will elute last
(c) b will elute first and a will elute last
(d) b will elute first and d will elute last
(6) If several compounds are present in a sample which is developed
on a TLC plate, a column of spots is seen on the developed plate, with:
(a) more polar compounds toward the top of the plate and less polar toward
the bottom
(b) more polar compounds toward the bottom of the plate and less polar
toward the top
(c) lower boiling compounds toward the bottom of the plate and higher
boiling toward the top
(d) lower boiling compounds toward the top of the plate and higher boiling
toward the bottom

7- TLC is generally used as a qualitative analytical technique for:
(a) determining the number of components in a mixture


146
Chromatography Course Dr Ehab Aboueladab-Lecturer of Biochemistry-Mansoura University-Branch Damietta


(b) checking the purity of a compound
(c) following the course of a reaction
(d) all of the above

8- In general, a compound will move further on a TLC plate with a:
(a) non-polar solvent
(b) polar solvent

9- Which of the following solvents is more polar?
(a) ethyl acetate
(b) hexanes

10- The melting point of a pure organic compound:
(a) is broad, having a range of 3 degrees or more
(b) is sharp, having a range of 1 degree or less
(c) has a range of 5.5 degrees
(d) varies according to atmospheric pressure


11- Calculate the Rf value for the following compound: spot, 3.0 cm;
solvent front, 10.0 cm
(a) 3
(b) 10
(c) 0.3
(d) 0

12- A beaker will be used as a "developing jar" in this experiment.
When the TLC plate is set in this beaker, the solvent in the beaker must
be:
(a) above the pencil line used to guide the spotting of samples
(b) deep enough to cover the entire TLC plate
(c) deep enough to come about halfway up the TLC plate
(d) below the pencil line used to guide the spotting of samples

13- Which of the following are flammable?
(a) hexanes
(b) ethyl acetate
(c) ethanol
(d) all of the above


147
Chromatography Course Dr Ehab Aboueladab-Lecturer of Biochemistry-Mansoura University-Branch Damietta




No Answer Explanation
1 C
The absorbent-packing must homogeneous and even throughout
the length of the column, otherwise the samples will not run at a
steady rate and evenly down the column. Bubbles, dry areas, and
unevenly packed areas channel the solvent unevenly down the
column; if this happens, the compounds do not move down the
column in discreet bands, but instead in streaks, and you will not
be able to collect fractions of pure samples. Also note that the
adsorbent, silica gel or alumina, is delicate and will not work
properly if it is dry.
2 b
Carotene is yellow-orange, as it states in the introduction to this
experiment in the Lab Manual. The spinach pigments are air-,
heat-, and light-sensitive. If you do not protect the dried column
fractions from air, light, and heat, they can undergo oxidation,
hydrolysis, and other reactions, leading to more spots than you
see on the column and/or more spots than in the original
mixture.
3 b
You must always begin with the least polar solvent, in this case,
hexanes. If you start with the most polar solvent, all of the
compounds will travel down the column very rapidly and
probably will not separate. (Remember: all compounds travel
faster when a polar solvent is used.) By using the less polar
solvent first, only the least polar compounds will travel rapidly
down the column. Once the faster-moving (less-polar)
compound(s) are off the column, you can switch to a polar
solvent to speed the elution of the slower-moving compound(s).
4 a
Too much of this polar solvent - methylene chloride - has the
same effect on separation as does using the most polar solvent
first. If a mixture to be analyzed by column chromatography will
only dissolve in a polar solvent, use as little of the solvent as
absolutely necessary to load the solvent onto the column
5 d
The order of elution of organic classes of compounds is (from
fastest to slowest): ethers (b) > ketones (a) > esters (c) >


148
Chromatography Course Dr Ehab Aboueladab-Lecturer of Biochemistry-Mansoura University-Branch Damietta


carboxylic acids (d).
6 b
TLC separates on the basis of strength of the adsorption of
compounds to the TLC plate (the adsorbent). The strength of this
adsorption is greater the more polar a compound, and therefore
a polar compound will not move as far up a plate as would a non-
polar compound. (Note that compounds are spotted near the
bottom of a TLC plate and travel up the plate with the solvent.)
7 d
TLC is a quick, powerful tool to check a reaction mixture or
solution of a compound. By determining the number and Rf
values which appear on a developed TLC plate, you will know
how many components are in the mixture, know if a reaction has
proceeded to produce product, or know if a compound is pure.
8 b
From the introduction, "generally, the more polar a solvent is,
the more effective it is at eluting both polar and non-polar
compounds.
9 a
Ethyl acetate is more polar than hexanes.
10 b
Pure compounds have a narrow melting point range, 1 degree or
less if the compound is very pure. A melting point range of 2
degrees or less indicates that a substance is pure enough for
most laboratory purposes.
11 c
The Rf is the distance traveled by the compound divided by the
distance traveled by the solvent.
12 d
The solvent must not be above the pencil line used to guide the
spotting of the samples. If it is, the samples will dissolve into the
reservoir of solvent instead of traveling up the plate.
13 d
hexanes, ethyl acetate, and ethanol are all flammable.
1-Discuss following in detail
a)Biospecific chromatography b)Gel Filtration

2-Write briefly on the following
a) Ion exchange chromatography: Principles, properties and uses
b) Affinity chromatography


149
Chromatography Course Dr Ehab Aboueladab-Lecturer of Biochemistry-Mansoura University-Branch Damietta


3-write about isolation and purification of biological molecules based on
ligand specificity
4-Discuss the following material for gel chromatography
5-write on the affinity chromatography
6-write on the following:
a) Thin layer chromatography
b) Ion exchange chromatography according to:
(1) Type of ion exchanger
(2) Preparation of the anion sorbent
(3) Methods are used for eluting molecules from the exchanger.
7- Discuss the following:
a)Classification of chromatographic methods
b)Adsorption chromatography
8- Write briefly on Separation techniques method used in chromatography
9- Write briefly on the following
a) Adsorption chromatography
b) different ways used in eluted the substances adsorbed on the
column
10- Write briefly on Separation mechanisms in adsorption chromatography
11- Write briefly on Common adsorbents and the type of compounds
adsorption
Chromatography
12- Factor affecting on separation of the compounds in adsorption
Chromatography
13- Write briefly on common adsorbents used for TLC
14- Discuss the following:
a) Thin-Layer Chromatography of Amino acids
b) Thin-Layer Chromatography of Carbohydrates
15-write briefly on chromatography techniques used to separates molecules
on the basis
of differences in size
16- Write briefly on determination molecular weight using gel filtration
17- Write briefly on Size Exclusion Chromatography
18- Write briefly on Gel Permeation Chromatography
19- Write briefly on Gel Filtration
20- Write briefly on properties of gels used in gel permeation (filtration)
chromatography
21- Compare between three types of polymers are principally used in Gel
Filtration


150
Chromatography Course Dr Ehab Aboueladab-Lecturer of Biochemistry-Mansoura University-Branch Damietta


chromatography (dextran, polyacrylamide, and agarose).
22-Definition of Chromatography and Principles of Paper Chromatography.
23-Differentiate between the use of a cation exchange resin and an anion
exchange resin in terms of whether the charged sites are positive or
negative and whether cations or anions are exchanged.

24-Match each term to one of the statements:
a) A chromatography configuration in which the stationary phase is
spread across a glass or plastic plate.
(b) A chromatography type in which the stationary phase is a liquid.
(c) A chromatography type designed to separate dissolved ions.
(f) A chromatography configuration that utilizes a fraction collector.
(g) The only chromatography type described by the letters GLC.
(h) One of two chromatography configurations in which the mobile phase
moves by capillary action opposing gravity.
1-Partition chromatography
2-Adsorption chromatography
3-Ion exchange chromatography
4-Size exclusion chromatography
5-Paper chromatography
6-Thin-layer chromatography
7-Electrophoresis
==========================================================
25-Tell what each of the following refer to: GC, LC, GSC, LSC, GLC, LLC,
SEC, GPC, DEAE cellulose and CM cellulose
25-Consider a mixture of compound A, a somewhat nonpolar liquid, and
compound B, a somewhat polar liquid. Tell which liquid, A or B, would
emerge from a chromatography column first under the following conditions
and why:

(a) A polar liquid mobile phase and a nonpolar liquid stationary phase
(b) A nonpolar liquid mobile phase and a polar liquid stationary phase

26-We have studied four chromatography types. One of these is partition
chromatography. Answer the following questions concerning partition
chromatography yes or no:
(a) Can the mobile phase be a solid?
(b) Can the mobile phase be a liquid?


151
Chromatography Course Dr Ehab Aboueladab-Lecturer of Biochemistry-Mansoura University-Branch Damietta


(c) Can the mobile phase be a gas?
(d) Can the stationary phase be a solid?
(e) Can the stationary phase be a liquid?
(f) Can the stationary phase be a gas?

27-Answer the following questions either true or false
(a) The stationary phase percolates through a bed of finely divided solid
particles in adsorption chromatography.
(b) The mobile phase can be either a liquid or a gas.
(c) The mobile phase is a moving phase.
(d) Partition chromatography can only be used when the mobile phase is a
liquid.
(e) Adsorption includes LSC.
(f) In partition chromatography, the mobile phase partitions or distributes
itself between the sample solution and the stationary phase.
(g) If the stationary phase is a polar liquid substance, nonpolar components
will elute first.
(h) Size exclusion chromatography separates components on the basis of
their charge.
(i) Gel permeation chromatography is another name for size exclusion
chromatography.
(j) Ion exchange chromatography is a technique for separating inorganic ions
in a solution.
(k) Paper chromatography is a type of LLC.
(l) Thin-layer chromatography and open-column chromatography are two
completely different configurations of GSC.
(m) It is useful to measure Rf values in open-column chromatography.
(n) Rf values are used for quantitative analysis.
(o) TLC refers to thin-layer chromatography

You might also like